You are on page 1of 63
As @>3, cothant ly | Ve sos 725) i.e. the voltage gradient at the line end is @ times the valuc of gradient corresponding. to the uniform voltage distribution (the voltage gradient corresponding tw uniform distribu. tion is V volt/metre). Fig. 7°60 shows the initial distribution of a surge voltage over a uniform windit of earth tribution with neutral. The voltage distribution ir uniform for a=0 but in the pre capacitances « may typically assume value of 10, and in that case, the voltage becomes nonuniform with most of the voltage dropped across.’ iact on of the line end of the winding, in which the voltage between adjacent turns becomes excessive and may Fequure additional insulation to withstand the high voltage stresses produced Final Voltage Distribution, The tail of the step voltage is equivalent to a sus- tained d.c. voltage of magnitude V after the all transients have died down and the system has settled to steady state c mnditions, ‘The voltage distribution therefore becomes v=V(a/1) This distribution is obtained with time t= 0. voltage distribution a unifo This distribution is identical to the one obtained with « =0. It is clear from above, that the final is a straight line giving a uniform voltage gradient in the winding. entieal . (See Fig. 7°61). vas = : $ 3 é 307 : & 3 gos 5 a Ratio il Fig. 7°60. Initial voltage distribution Fig. 761. Oscillations in winding subjected to voltage Me PO ith earthed neutral. surge. Intermediate Distribution. The transition from initial to final voltage distribution is always accompanied by oscillations on account of inductance and capacitance of trans- former winding. ‘There is a constant interchange of stored energy between capacitors and winding inductance resulting in complex oscillations at a variety of natural frequencies, ‘As ar-sult of damped transference of electrostatic and electromagnetic energy during complex oscillations, all parts of winding may be severely stressed (i.e have large voltage Fents) at different instants of time as shown in Fig. 7-1. Initially concentr.tions of eoitage may appear at the line end of the winding ; during tran.itional period concentrar Hons may appear at the neutral end whilst voltages to earth considerably in excess of the incident surge may develop in the main body of winding. Therefore, reinforcement of end turns of the winding are oflittle avistance as far as the protection of windings against surge voltages is concerned, 394 . ELECTRICAL MACHINE DESIGN + 7-40,1, Surge Protection. In a general case, under steady state conditions, equal voltages fare induced between turns and consequently, ideally, equal amounts of insulation ate required between turns. To utilize this uniformly disposed insulation to best advantage, the voltage appearing between turns throughout the winding under surge conditions should also be able to approach this ideal, in which case oicillation voltages are completely elimina- ted, the initial distribution, like the final must be uniform. Before the effect of surge voltages was clearly understood, it was a universal practice to reinforce the insulation of a few turns (say 5%) on the line’ end of the transformer to withstand the impulsive voltage gradients. However, this results in decrease in the series capacitance resulting in increase of a and hence unequal voltage gradients. The use of reinforced turns at winding ends, instead « f mitigating the trcuble, intensifies it. External surge absorbers may be connected between the transmission line and the transformer terminals to reduce the steepness of wav. front and to dissipate some of the energy of the surge wave. However, .he real solution to the problem of surge voltage distribution lies in design- ing the winding in such a way that the voltage distribution is miore or less uniform and no part of the win ing insulation is unduly stressed. The initial distribution of voltage is determined wholly by the capacitance network. ‘The voltage distibution depends upon « which is equal to ~/G, Consequently two cir- cuit elements are available for controlling and improving the initial response. It has been stated earlier that when a==0 (i.e. when the shunt capacitance C,=0 or when series capacitance Up=00), the initial distribution is uniform and is coincident with final distribution. Thus, can be’ safely concluded that the distribution can be improved by decreasing shunt cap: tance and/or by increasing the series capacitance. The methods adopted for securing uni form voltage distribution are : 1, Shielded Windiags, The basic principle of protection of transformers rated at 110 kV and more involves the arrangement of an additional: capacitance network con- nected to the line input terminils in order to neutralize thé winding to earth capaci- tance. ‘An electrostatic shield is provided along the axial length of h.v. winding as shown in Fig. 7°62. The provision of the electrostatic shield introduces capa:itances between the Capacitive tine is Fig. 7°62, Winding with static shield. ‘TRANSFORMERS 395 li ind the winding in a graded diminishing proportion asin a transmission Tine insulating string, thereby helping to neutralize the winging to ground capacitaness The use of the shield permits the shunt currents tu flow directly ftom the line, thereby enabling the series currents to have the same value in all the sections. Hence a uniform voltage distribution is obtained. The surge voltage distribution with and without electro static shields is shown in Fig. 7°63, the figures indicaing percentage equipon ntials. ‘The surge prote Ad involves the following procedure : (j) The shield is provided on the six terminal coils (for 110 to, 220 kV) at increa.ing distances. (ii) A capacitive ring is placed over the winding to equalize the voltages across the turns of the input coil. The ring (or collar) is of pr “sboard with copper or aluminium foil and is securely insulated. The capacitive ring is connected to the line terminal of the winding. 2. Centre point disc winding. In this method, the high voltage dise winding is divided in two halves which are connected in parallel. Each half starts from opposite end and finishes at the centre point. ‘The centrepoiut becomes the h v. terminal of the winding (Fig. 7°64). ‘The hav. winding is provided with radial and complete axial shields and the arrange. ment results in uniform initial and final distributions. 3, Interleaved wind! Tw> normal disc coils are shown in Fig. 7°05 (a). If the order of the turn interconnection is changed to have interleaved coils as in Fig. 7°65 (8), the interturn or series capacitance Cy increases thereby reducing «. Therefore, voltage distribu- tion becomes more uniform. of transformers by an clectrostatic sh Windings of interleaved double disc type can withstand impulse voltages better than the non-interleaved type. Partially interleaved windings are also used with interleaved coils only at the ends of h.v. windings. 4. Layer type windings. Recent developments VEUTRAL in the design of high voltage transformers emphasise the difficulties of obtaining adequate impulse strength with TT continuous disc type winding. Modern trend is to use 164, Centre point layer type of winding as shown in Fig. 7°66 winding. ‘The high voltage winding is divided into concentric layers, separated by oil ducts, ste Ng " v Core 4 u © ® - Fig. 765, Normal and Interleaved coils, Fig. 7 66, Layer type winding.. 993 ELECTRICAL MACUIINE DESION cach ciminishing in length. | Two shielis are used, one atthe line end and the other at. the neutral end. Oa account of the provision of shields on the two ends, it is possible to neglect the capacitance between winding and earth (Cs) in comparison with capacitance between adjacent turns (C, ‘Therefore, « is umall for these windings and hence the voltage di uniform, Fig. 7°67 shows the voltage distribution in a layer type winding Tei cloar thatin a layer type winding the voltage distribution initia'ly and under transient conditions is almost uniform. ‘The layer type cf winding is thus lighting proof. ibution is almost ‘The same conditions exist in the windirgs of shell. type power transformers because the capacitances between large flac coils considerably exceed the capacitances rclative to earth ‘and therefore, the initial asd the final voltage distributions practically coincide. LT im wally of petentols| § oF initial o Getrbution 7 Capecitar ye eal radient i BI . os os 075 10 : Ratio, x/! Fig, 767, Initial vol distribution and peaks of potentials in ayer pe winds ‘T41, Window Space Factor. The window space factor is defined as the ratio of copper area in the window of the total window area. It depends upon the relative amounts of insulation and copper provided, which in turn depends upon the voltage rating and output of transformers. The following empirical formulae may be used for estimating the value of window space factor : - Kom 10/(30+4V) a= (7'99) where kV is the voltage -of h.v. winding in kilo-volt. The above formula is for trans- formers of ratings between 50 to 200 kVA. Space factor is larger for large outputs and smaller for small outputs. For a transformer of about 10.0 KVA rating Ke=12/(90-+LV) and for transformers of about 20, EVA rating Ko=8/(30+KV). The values of space factor for intermediate ratings can be interpolated. 7-42, Window Dimensions, The leakage reactance is affected by the distance between adjacent limbs. If tLis distance is relatively small, the width of the winding is limited and this must be counter balanced by increasing the height of the winding. Thus the windings are long and thin. This arrangement leads to a low value of leakage reactance, If the height of the window is limited, the width of the window has to be increased in order to accommodate the coils. This results in short and wide coils giving large value of leakage reactance. ‘The height and width of the window can be adjusted to give a suitable arrangement of windings si uno to give a desired value of leakage renctance, ‘TRRNSFORMERS ‘The area of the window depends upon total conductor area and the window space factor. total conductor area ‘Area of windew dw~-\indow space factor = 2a for single phase transformers eA7-2N) = 4927» tor three phase transformers (7-28) Area of window Aw=height of window X width of window=Hex Fw ‘The ratio of height to width of window, Hu/ Wu is between 2 to 4 Assuming a suitable value for ratio Hu/ Ww, the height and width of window can be calculated, 743 Width of window for optimum output. Let D be the distance between adjacent limbs as shown in Fig 7°C8. Now, De=width of icon-+width of bare conductors-+width of insulation and clearance, Let m be the space occupied hy insulation and clearance ete. along the width <. Width occupied by copper plus iron D’=D- m. Width of bare conductors in window =widih occupied by irdn plus conductors— width occupied by iron=:D'—d. Let 8 be the output in VA of transformer per unit height of window, c. S=Ee TnI where Ta=turns per unit height. Now Bi=4'44 f qm 44's f Be At==4 44S Bm bel. For a fixed value of frequency and flux density, H+ is proportional to d?. Now [Ts=mmf per unit height=8 a Tr where asvarea of each conductor and Securrent density Now aT=height of conductors x width of conductor: width of conductors (as height is unity). IT. is proportional to (8X width of copper in window) _ or ITs is proportional to (D’—d) for a constant value of current density. s. S=Ad* (D'—d)m Ad? D'— Ad? where A is a constant. In order to determine the maximum output for a given value of D’, 8 is differentiated with respect to d. or SBnvav'e- s4ét, For maximum output, SP—0 * 2dD'dm3Ad? 1, Dl Sd. Now D=D'+w. The value of m can be taken as 0'2d with normal designs. or D=i7é ’, "The width of w'ndow which gives the maximum output is We=D—d=0'7g ‘744, Design of Yoke, The of the in taken as 15 to 25 percent larger that of core for nSings tlie lead tech ‘Ths redoes the falue of fut denaiey obtaining in the. i there is reduction in the iron lomes and the manetis- than ‘current. “For aan formers using cold rolled grain oriented steel in the area of yoke is ’ ‘equal to thit of the core, 398 ELECTRICAL MACHINE DESIGN The svetion of the yoke can either be taken as rectangular or it may be stepped. The yoke sections are shown in Fig. 7°15. In the case of rectangular section yokes, the depth of the yoke is equal to the depth of the core. This depth of the core is equal to the width of the largest stamping when square or stepped corcs arc used. For example in the case of cruciform core (Fig. 7°52), the depth of yoke is equal to a. For rectangular section yokes, Area of yoke Ay-=depth of yoke x height of yoke==Dr x Hr where Dr=wilth of largest core stamping=a Ar=(1'15 to 1°25) Ags for transformers using hot rolled steel =A for transformers using grain oriented steel. 7-45, Overall Dimensions, When dealing with overall dimensions in transformer problems, refer to the following details and dia- granu. ata t a=width of largest stamping, Le __ D=distance between centres of adjacent limbs, 1 d=diameter of circumscribing circle, | t We=wilth of window, i : He=height of window, length of limb, fa Hr=height of yoke, H=overall height of transformer over yokes ke cor overall height of frame, Pd W=length of yoke=overall length of frame. ze We have the following relations for single | ‘ohase core type transformers (Fig. 7°69). 7 y, t ~ 3 ~ ho 4 D=d+ We, Dr TT Widin over two H=He+2Hr, W=D+o, Fie. ee hee core Pe Width over two limbs=D-Fouter diameter of h.v. winding, Width over one limb=outer diameter of h.v, winding. We have, for a 8 phase core type transformers (Fig. 7°70). Ded+We; Dra; H=Het2Hr ; W=2D+a, Width over 3 limbs=2D-t outer diameter of h.v. winding. Width over one limbssouter diameter of h.v. winding For single phase shell type referring to Fig. 7°71 Dre, Hr=o, W=2Wet4o, Hm He t20, Example 7:1. Estimate the reduction in volume, expressed Percentage solume of (i) iron core ll) conductors, iva transformer then hat olla eiteon eet li TRANSFORMERS: a8 Ydesceree . Depah over winding Width" over three limbs Fg. 7-70, ‘Three phase core typ transformer. Fig. 7-71, Single phase shell type transformer. tions worked at a flux density of 1°2 Wh/m? are replaced by colt Folled grain oriented silicon steel laminations worked at 1°8 W'b/m*. Circular coils are used «ind the thickness of windi-g te negligible aa compared with diamrter of circumscribing cirele. Phd langth of fluz path, area of conductor und the number of turns are the same for both eases. Solution. Let Amand Ay be the net iron areas with hot rolled and cold rolled | aminations respectively. ‘The net iron area is inversely proportional to tux density. a Ate=(1°2/1°8) Acn=0°667 Avr Hence, redaction ir net iron area=(1—0°66") X 100—33'3% Letds and de be the diameters of circumscribing circle with hot rolled and cold rolled laminations respectively. Diameter of circumscribing circle is proportional to +/4y. ” dem O667 = dy =0'817 da. The length mean turn isd, Therefore, length of mean turn ot conductors wound on cold relied steel is 0°817 times that of conductors woun | on hot rolled steel. Since the num- ber of turns and area of each conductor are the same in both cases, the reduction in volume of conductors with the use of cold rolled steel is (1—0°817)X 100 18°3%. Example 7'2. Show that the output of a 3 phase core type transformer ia : Q=5'23 f Ba Hi? Hex I0* vom cthereSprquncy, Hs, Bacomasiam fluc density, esamatr of core, Hamantic ial gradient in lim), Alm ; sgl of tin Porndow 400 ELECTRICAL MACHINE DESIG Solution. kVA output of a three phase transformer Q=3EI x 107 3X 4°44 f Um TIX 10-*=3 x 4°44 f Bm Ac TTX 107%, In a three phase core type transformer each limb has one primary and one s‘condary winding wound gn it and therefore total mmf over one limb=27/, ‘ ‘ . 5 mmf Magnetic potential gradient Hee or imp at ie rate Also Ac=(n/4) 2 Substituting the value of 4s and °F in the expression for Q, we have 23 f By Ha? Hw X10-* kVA. Qa3x 444 f Bax atx Bex 10-s Example 73, Calculate the kVA output of a single phune truneformer from the follow. ‘ing data: core height agig, ‘Sameter of circumscribing circle _ 554 Gatance between core cenlves > ' distance belween core centres g net iron area “07 area of eircumneribing circle current density=2'3 Aymm?, window space factor =0'27, frequency = 50 Hz, Sux density of core=1'2 Wo/mt, distance between core centres=0'4 m, Solution. Distance between core centres D=0'4 m. Core height (window height) Hw=2°8X0'4=1°12 m. Diameter of circumscribing circle d= 0°56 X0'4=0 224 m. Width of window We=D—d=0'4—0'224=0'176 m Area of window Aum HeX We==0'112X0'176=0'197 m* Area of circumscribing circle ia=(7/4) d®=0'0394 m* s+ Net iron area Ac=0°7 X0'0394=0°0276 m* From Eqn. 7°6, for a single phase transformer Qm222 f Bm Ku 8 Aw AtX107? kVA Sm 9'22 50 1°2 0°27 2°3 x 104% 0'197X 0°0276 x 10450 LVA. Example 7-4. Determine the dimensions of core ond yobs fora 200 BVA, 60 He single transformer. orwciform core is ith distance Heads coast 1 6 times he width of cor laminations, dsewne. wollage per harm 1d Ve not sma fea density 11 Wom’ window, space factor 098, current density 3 Alt, and docking factor=0°9..The net iron area is 0°56 din a ‘core where dis the diameter of circwme. ‘eribing circle, Aleo the width of largeat stumping te 0°85 4. Solution, Voltage per turn Ey=4'44 Oume4'44 f Bu As Diameter of circumscribing circle dm 4/ dg0S6=+/0'0573]0'0~10'32 m, Width of largest stamping #095 d =0°85 x0'$2=0'272 m. Distance between core centres D=1'6 o (givan}=r1°6X0'272—0°435 m. ‘Width of window Wem D—d—0'43$-~8 00115 m. From Eqn. 76, for a single phase transformee, ‘TRANSFORMERS 401 Q=222f By Ku 8 AwAiX 10° . 200=2'22 x 50. 11 X 0°32 3X 10" AwX 00573 ¥ 10°* Window area Ay='0295 m? *. Height of window Ife=0°0298/0°115=0°26 m. Using the same stepped section for the yoke as for core Depth of yoke Dy=a=0'272 m and height of yoke Hy=0'272 m. Referring to Fig. 7°70, Overall height of frame H =Hw+-2ily=20-+2 x 0'°272=(" 804 m. Overall length of frame W=D+a=43'540-272=0 737 m Example 75. Calculate approximate overall dimenrions for a 200 kVA, 6600/440V, 50 Hz, 3 phass core type transformer. The following data may he assumed : emf per turn 10 V5 mazimum fluz density 1-3 Wolm? ; current de overall heightoverall width ; alacking facior=0'S and ‘Almm? , window apace factor=0°3 ; Une a3 stopped care. For a theee stepped core. Width of largest stamping=0'9 d, and Net iron area=0°6 d* where dis the diameter of circumscribing circle, Solution. Net iron area Am ggg 7 Diamster of circumscribing circle d= 00 47/0 width of largest stamping o=0°9X0'24=-("216 m Using a 3 stepped section for the yoke Height of yoke Hy=a=0'216 m, depth of yoke Dy=a=0'216 m. From Eqn. 7'10, for a3 phase transformer, Q=3'33 f Bm Kw 8 AeAix 1079 or 200=3'33 x 50X 1°3 x 0°S x 2'5 x 10* x Aw x 0°0347 X 107 or ” dow area Au=0°0355 m* or HeX We=0°0355 m® The given condition is, overall height=overall width or H= WF Now, referring to Fig. 7-71 H=Hut 2Hy=Ho+2X0'216—Het 0432 W="D+a=2( Wotd)+a—2Ww+0'48+0'216=2Wo+0'696 As H=W, we have: He+0'452=2Ww+-0°696 He=2Wo+0'264 2 (2 e+0'264) Wo=0°0355 or 2Wut-+0°264 Ww—0°0355—0 width of window Wu=0°083 m and height of window a =0°428 m ‘Thus the dimensions of core are : Distance between adjacent core centres D= We+¢=0°323 m. Overall height H=Hu+2H,=0°86 m. Overall width W'=2D-+a=0'862 m. Example 76, The ratio of fluz to full load mmf in a 400 kVA, 50 Hs, single phase core type power transformer is 24X10". Caleulate the net iron area and the window area of the transformer. Mazimum fluz density in the core ia 1'3 Wb/m?, current density 2°7 Alem and window space factor 0°26. Aleo calculate the full load mmf. Solstion, From Eqn. 7°13, Kad £44 f (OnlAT) 10) FEAKGOXD AXIO 100732 Voltage per turn Eve K*/Q=0°732-/ 400—=14'64 V. Lc IRICAL MACHINE DESIGN E 14°64 Flux n= 544 f = 44x =0°066 Wo. Net iron area Agate, Window area of single phase transformer yp pe 0.0777 eit, 2:22 x DONT SRO TOXZTX OKO USOT RIO 8 00777 On 0-066 Full load mt AT = 5-7 Op 9-4 gre = 27500 A Example 7:7. Determine the main dimensions of the core, the number of turns and the cross-section of the conductors fora 5 VA 11000/400 V, 50 Hz, single phare core type distribution transformer. The net conductor area inthe window is 06 limes the net rross- ection of irom in the core. Ansumr. a aquare cross-section for the core, a fluz density 1 WO/m!, @ current density I'4 A/mm, and a window space faster 02. The height of window is 3 times ita width, Solution, Given that : Net conductor arcas=0°6Xnet iron area or — Kwde=0'6 Ai i 06 4206 2. Window area Ae= a" domo Ar=3.At From Eqn. 7°6, for a single phase transformer, Qe 222 f Bm Ke 8 Ay Aix0-® = 222K 50X10 X0 2X 14x 10°XS Aex dex 10-8 or net iron area Ai=0°00732 m?. Gross iron area Agi = =0 00814 m*, we Width of core a=+/0°00814 0°09 m. Gross iron area provid d=0 0081 m2. Net iron area provided=0 00729 m#, Window area Aw=3 x100729=0-02187 m?, Height of window Hw=3We — . 3Wet=0'02187, But Hux We= Aw, or width of window W™#0'085 m, and height of window H»--30°085=0'255 m. The yoke hrs the same gross aren as the core, Gross arca of yoke 4y=0°081 mt, Depth of yoke Dya=0'09 m, «, Height of yoke Hy= 700 0°09 m. Flux @u= Bu Arm 10 X 72°9 X 10-8 7°29 x 18-* Wh. Voltage per turn Br~ 4°44 f m= 4°44 x 50 X 7°29 x 107 1675 V. is — Primary volta w= 11000 5 oan Primary turntee errary valigee Ximcondary turnsse “ope x 245-—6765, (The turns of the low voltage should be calculated first and that of high voltage after- wards by using the voltage ratio). secondary voltage 400 Secondary tums Tore cTage per tum "1625 Primary winding current Tye OO 0055 A ‘TRANSPORMERS ws Area of primary winding conductor ap= =0°384 mmt, ‘Using circular conductors, diameter of primary conductor = 1°S24X4/m=0°642 mm 5000 _ Seconlary winding current Je= o> = 12 5 A. Area of secondary winding conductor +=" ‘Using a square conductor 3x3 mmt. Area of secondary conductor 1=9 mm? Overall dimensions of core— Referring to Fig. 76: Distance between core centres Dena-+ We=0'09-+0°085=0'175 m. Length of frame i =D+a=0'175+0'09=0'265 m. Height of frame H=Het 2Hy=0°55+2X009=0'435 m. Example 7:8. Calculate’ the main dimensions and winding detaile of a 100 ki'A 2000/400 volt, 50 Hz, single phase shell type, oil immersed, self cooled transformer. Assume : Voltage per turn, 10 V flux density in core, I'1 Whim? ; current density, A/mm? window space factor, 0°33. » The ratio of window height 4o window width and ratio of core depth to width of centri Timb=25. The stacking factor is 0'9. . ‘ j E 10 Solution, Net iron area Arm rg rae FqaxSORTT is A204 _ 5, . Gross iron area Ag= a 070553 mi =0'441 mt, Referring to Fig. 7-71, z =2'5 (given), We have, gross iron area Agi=2aXb 2°5 (2a)*=0°04555 or width of central limb, 2a=0°135 m. Core depth b==2°5 x 0°135==0°3475 jn. ~ The yoke carries half of the flux in the central limb. Assuminggfhe eame flux density in the core as in the limb, the area of yoke is equal to half the area,pithe cemtral limb. Gross area of yokedy= TONS 922775 10-4 mt, Depth of yoke Dy=b=0'3975 m. Height of yoke ges ET TT 00675 m. The side limbs carry half of the flux in the central lifib. Therefore, the width of side limb is half of the width of central limb. Width of side limb a=-0°0675 m. From Eqn. 7°6 for a single phase transforgye Q=2'22 f Bn Ke As eX 10? : 100==2°22 x 50x I'1 X0°33 x2 108X0°04I X Aw X 10 We have, Hox Wem0'0803 and Bt a, Window area 4e=0'0308 mm? ELECTRICAL MACHINE DiSIGN or 31'e'=303X10-f, Thus width of window Tl Height of window He=0'3 m Referring to Fig 771 Overall height of frame H=He+2Hy=0° +2X'.675~0'435 m. Overall length of frame W=2W'w +4a=2X0 1 +4%0°0675=9 47 m. Overall depth of frame=6=0°33; om Windii HLV. winding tures 7, = 9 ), LV. winding turns T= AO ona00, 0 100% .0.0 FLV, winding current Jpm!3oy5-—=50 A. 50. HLV. winding conductor area a=” —=25 mm*. 100 1000 = 400 250 . Sp ol-5 mt, V. winding current =250 A. LV. winding area Example 79, Calculate the core and window areas of a 400 kVA, 50 He, single phase, core type power transformer. ‘The fullowing data may be assumed : Ratio of weight of iron to weight of copper=4, Ratio of length of mean twn of copper to Lngth of mean flur paih=0 6; mazimum flur density=1 5 Wh'm* ; current density 22 A/mm ; density of copper=8'9%.108 kg/m ; denaity of iron=7°8 x 10° kg/m, copper space factor=0°12. Solution, From Eqn. 718, 1, Lm te y 198) Om( aap: FE xi0e)"=( From Eqn. 7°17, net iron area 1 n 16x (sq age 4 J" =0'0478 me 5 89X10 Ia! Tap XO5x Testo 1) =16 BOX 15X22 x 10® From Eqn. 7°6, win ‘ow area Ae: ss ea 2297 Bm Ke BAx10* 400 = $0183 mt. PPIX BOXTSXOTINZ IX 1O°XO 0498 1 3 mt Example 7-10, Calculate the ratio of weight of iron 10 weight of copper, net iron area, voltage per turn and the constant K ‘where vollage per turn Ex=Kr kVA) for a 500 BVA, 50 Hz single phase co + type powrr transformer for ($) maximum efficiency to occur at 90 percent of full load (ii) minimum cost (iit) minimum weight (iv) minimum colume. Assume ; maxi. mam fluc density 15 Whim; current density=275 Almm*; resistivity of copper at 18°C=2 1X10 Qm; density of iron=7°65x 103 kg/m? ; density of copper —8 9x 10° kg/m? ; ratio of eprcific cost of coppr to specific cost of iron for of mean length of turn of windings to length of flue paih=0'S ; stray toad loss=10 percent of full load copper lose ; iron loss per kg for 1.5 Wlm?=1 23 W Amnme an ezira loss for Jointa=20 parcent of total iron loss. ‘TRANS FOKMERS “5 Solution. Spevilic iron loss pee "2x 123=1 475 Why. Bp _ (27S KI EXBTXIO Loy Box a> BIS ee 178 Wikg. Specific copper loss including stray load Joss pes= "1% 17°82 19'6 W/kg. Specific copper less= Ratio of weight of iron to weight of copper to give maximum efliviency at 90% full load. (iP ogy 196 ciys Tee pe SOM Taye 1057. ; ¢ ty ga)? Now, -( TE hw ) Lory: BYX1 Lg ~( XOX y ppxeque x10) 16 iron area dee( 2. GLY of — 50 ays VF Net iron area As o( sibs = 6 Speseyaw «1075 =0'0819 m*. Voltage per turn Br=4'44 j By Ave d44x50% 1°5 x(0H19=97 97 V. Be 27 3 Constant Ke =1'22. Ve 4500 (i) Ratio weight of iron to weight of copper to give minimum cest is (ledpeefensed 5 tn wl Net iron area Ai=16 (antttmar ) a1) 05 me Eyed x 50X15 X05 16°65 V avd pm 885 Lo-745 V0" (ii) For minimum weight, Go 1 a Aj=0°025 m2, Be= imum volume Ge _ 185. Gs ge 89 ‘ Ac=0'023 m? Bem 77 V and K=0'344, Example 711, A h.v. disc winding has 9 coils each separated by am oil duct aa shown in Fig 7°62 page 394. Buch coil has 10 turns as shown Compare the series capacitance. of the interleaved dise coil pair aa ahown in Pig. 7 65 (b) page 3:95 with that of normal pair es oom in Fi. 765 (0) Solution, (3) Normal Coil. Let E be the voltage per coil pair and Ci the turn to turn capacitance. s+ Voltage between adjacent turns £;=Z/10 as there are 10 turns in each coil. <1. Electric field energy between successive and adjacent turns 1 E\ waz (i) There are 8 complete pair of ducts (as there are 9 coils) and therefore the total electric field energy WB x $C (Z/10)*m0'04 Cue* oli) 34.V, and K=0°375 0°86 0 ELECTRICAL MACHINE DESIGN Let Cx be the total effective series capacitance between turns | and 10. Total electric Geld encrgy Week Cr, B* oli) Equating (i) and (i), we have : x,—0'08 Cr. (i) Interleaved coil ‘The geometry of the turns is unchanged and therefore Ce remains the same. However, the interleaving effect increases the voltage between adjacent (but no longer successive) turns to 5 Ei (eg turn 1 t0G, “to 7ete.) or to4 6 102 and 7to etc) Total electrostatic ficld energy Werad Cs [452 /10)2+4142'/10)3] 0°82 Cab ii) Let Cx, be the effective series capacitance in th \d encrgy=# Ong Bt |, the effective series capacitance Cxz=1°64 Cr Henee, ratio of effective series capacitance for interleaved and norm i coils is Cy 164 Logs case. ‘Therefore, a for the interleaved cuils is 1/-/ 20°3=0'22 times that for normal coils. Hence vie voltage distribution in interleaved coils with surge voltages is much more uniform than that in normal coils. OPERATING CHARACTERISTICS 746, Resistance of winding. Let Lmip, Imis=length of primary and secondary windings respectively, m ; tr», tu=resistance of primary and secondary windings respectively, m. mp lat and yng dee Total PR loss in windings Pom lfrp+Itrs ‘Total resistance (per phase) of transformer referred to primary side Peng (18 eres 22) ; Ryne (EY nmr (Fn +730) Per unit resistance one eATS1) 741, Leakage reactance of winding, primarily the estimation of the distribution of leak the primary and the secondary windings. The distribution of the leakage flux depends upon the geometrical configuration of the coils and the neighbouring iron masses, end also on the permeability of the latter. The estimation of leakage reactance is ge flux and the resulting flux leakages of ‘The most common arrangements of the core and the windings are of those of core and thell types as shown in Fig. 7°72. Fig. 7°72 (a) thows the case of cylindrical core type of win- dings of equal axial length. The leakage field is mainly [packed into the space between the windings (i.¢. into the duct) and runs parallel with the core for nearly the full Tength of the coils. The distribution of leakage flux in the case of shell type of transformers having sand- wich coll it thown in Fig. 772 (6). The fl ie mainly packed into the ducts between (@) Concentric Sandwitch colts winding and runs parallel along the width of ___ winding o the cols in this case, : Fig. 7-72, Leakage fx in traodormer windings, ‘TRANSFORMERS 407 In both of the above two cases, the field is sufficiently symmetrical and geometrical and therefore considerable simplifications can be used to evelve usub‘e approximete mathe- matical relationships. TATA, Leakage reactance of core type transformers. The arrangement of windings of a core type transformer is shown in Fig. 7'73. ‘The problem of calculation of leakage flux and consequent leakage reactance is greatly simplified by making the following aaaumnption : 1, The primary and the secondary windings have an equal axial length, 2. The flux paths are parallel to the windings ulong the axial height. 3. The permeance of the leakage flux path external to the winding Le is taken to be so large as to require the expenditure of a ngligible mruf ie. whole of the winding mmf is expended on the length of flux path of length Ly ; .is path being entirely through air. Therefore, in fact, the assumption is that the mmf requircd for iron pirts is negligible, 4, The primary winding mmf Zyly is equal to secovdary winding mmf Ls There: fore, the magnetizing mmf and hence magnetizing current is equal to zero. The total mmf AT=IpTo= i. 5. Half of the leakage flux in the duct links with each winding. 6. The lengths of mean turn of the windings are equal 7. ‘The reluctance of flux path through yokes is negligible. ‘Therefore, the reluctance of yoke does not affect the flux distribution. 8. The windings are uniformly distributed and therefore the winding mmf varies linearly from zero from one end to AT at the other end. Three leakage flux paths ®, ©, and ®, are shown ®y,, are the leakage fluxes in the primary and the secondary windings respectively while . is the flux throu h the duct. Let? L,=mean circumference of duct, ‘Le=axial height of windings, s=radial width of primary and secondary indings respectively, a=:width of radial duct. ‘The flux leakages of the windings can be found as under. Conductor portion. Consider an infintiesimal stiip of width dz at a distance 2 from the edge of primary wiading along its width. fmf acting across the strip by ~E . Permeance of stip no 2 ae 1. Flux in the strip 1) 2 Imi (LT ” Fp Xm de ap, 2. as. Fig. 7-73. He Zloty ae ‘ehution ia core ype tnsonmest” ‘This flux links with (2/by)7> turns. ve Flux linkages of the strip stim Bint, Laex Etre te (4) és 408 ELECTRICAL MACHINE DESIGN Hence, flux linkages of primary winding duc to Aux in the strip by Ime nat Vane, le 1,22. Te PW, ye HoT eS tm [ahe=ne Dact portion, Mmf acting across duct=J7y. Peameance of duct=; Flux in duct @e=/ot's po amps Jnr F-, Half of the duct flux links with each of the two windings or duct flux linking with primary winding=—>- jo Jy2'y fa. ‘This flux links with the entire primary winding. Flux linkages of primary winding due to duet flux dem Lally a Ty Hence, total flux linkages of primary winding Wattle pe ate ( Lnty F thle +) “The above expression is siaplfied by assuming Iim= Lmip= Le ‘Leakge inductance of primary winding = ay Leakage reactance of primary winding 6, ym te foe st (4+4) +773) ‘The leakage reactanee of the secondary winding can be similarly calculated by assum- ing Law Le= Let. c. Leakage reactance of secondary winding +774) Leakage reactance of secondary winding referred to primary side wan (Se) atte te) Total reactance of transformer (per phase) referred to primary side othe ) ef fon ta (. aye) Per unit reactance eas — 477 =: AT To at tet) nnn FE +776) ‘TRANSFORMERS 409 as T,|[Vy=Ec and IpTy=A7' where AT is the total mmf per limb of cither coil. In some cases, one of the windin,s, usually the hv. is split up in Fig. 7°74. The leakage reactance of transformer per phase) referred to primary side in this case car be written as Kem te fut at beth the + ) wl PTD) to «wo paris as shown, as the p.u. reactance conan foe A E(t ) soe 778) where a close regulation of voltage is essential as ir distribution transformers used for purely domestic pu poses, it is necessary to make the leakage seactan small so that the voltage remains within 5% of the rated value, ‘The obvious way to achieve this is to sandwich the inv, winding between two sections of I.v. winding as shown in Fig. 775. For the purposes of analysis the actual arrangement ofthe windings on each leg can be considered ‘as consisting of two groups o} connected in series, with each roup ‘comisting of half the total number of Fiz. 7-74. Minf distriburion in turns per phase for each winding. transformers with hv. winding From Eqn. 7°76, total leakage reactance of transformer (per phase) referred to primary side iL TT, \* a be Snmtetee (FY (ot Be and the total leakage reactance of transformer (per phase) referred to primary side Int Spencer fuels BEC (7-79) Mat, in core type transformers with h Fle Fading sandwiched betweea wo sections of .v wioding. a0 ELECTRICAL MACHINE DESIGN Comparing Eqn. 7°79 with Eqn. 7°75, it ean be safely concluded that the leakage reactance is reduced to nearly one fourth by subdivision and interlacing. 7472, Leakage reactance of sandwich coils. The idealized flux distribution in shell type transformers using sandwich coils is shown in Fig. 7°76. Let the h.v. winding have n coils. Each of the m coils is sandwiched between two coils of l.v. winding. This requires the l.v. winding to have two half coils, one ‘at each end of the winding and a— full coils spread in between. Each half coil of l.v. winding contains half the number of turns of that of a full L.v. coil. Thus, if there are w coils in the h.v. winding, there will be n—1 full coils and 2 half coils in the L.v. winding. ‘The half coils have half the turns of a full coil. The win- ding can be considered as consisting of 2n units connected in series with each unit consisting of a half Iv. coil and a half hy. coil. Each of these units can be treated on the same basis as that of cylindrical concentric winding. The width of the coil, W, can be taken analogous to axial length Ls, of the cylindrical windings. _ Leakage reactance of each vo primary side with analogy to Eqn. 7 Xuntn tad 7 t, Se (0 a att ) as Tylinenumber of turns in each hal’ coil of primary winding. . J Total reactance of transformer (per phase) referred on the primary side Ty y fat ( byte ) fat a4 per phase) referred Fig. 7°76, Sandwich wining. Xp Qn x In fae (# an Ww 6 Lm Pet (4, bythe one ae A (e+? te.) o(T80) ny AT Lm be: Per unit reactance eye —lte, AP | e( ap tents -AT81) Example 712, Phe current densiliew in the primary .and secondary windings of a 10:1 and transformer are 2°2 and 2 1 A/mm reapectively: The rativ of transformation i the length of mean turn of the primary is 10 per cen! greater than thal of the secondary. Cal- culate the resistance of the secondary winding given that primary winding resistance is 8.Q. Solution, Resistance of primary winding Lp Imp _, LoLmip. romp ot = Similarly, resistance of secondary winding rem (Ws Inte Bs e Te TRANSFORMERS aul nee oo B YC EYEE) me) CBM Example 7:13. 4 300 LV.A, 6600/00 V, 50 Hz, deltiiatar 2 phase evre type transe former has the following data : width of hv. winding=25 mm ; width of Lv. 16 mm ; height of evite= length of mean turn=0'9 m ; hv. winding twrne=83 width of duct between he. and 1.v. windings=15 mm (a) Catoulate the leakage reactance of the transformer referred to the hiv. aide (b) If the Lv. coil ie eplit into two parts with one pat on cach side of thx hv. coil, cal= culate the leakage reactance referred (0 the h.v. side. Assum that there isa duct 15 mm’ wide between h.v. winding and each part of | v. winding. Sm; Solution, Leakage reactance referred to the primary side L b, stent fo 2) = 28x 50 x 4 x 107? X (830) B( ois PEE agg (b) The L.v. winding divided into two parts, one on each side of hiv, win ing and therefore Eqn. 7°79 is used. omfg TE a+ oH ) ex go xanx 10" x (es0rx 72 ( omi54¢ SEEN OG )-536 0. Example 714, 1, 100 VA. 2000/40 V, 60 Hz, sing'e phase abril type transformer has sandwich coils. ‘There are two fuil h.v. co lt, one full Lv. coil and, two half Lv. coils Calculate the value of leakage reactance referred to h.v’ wide, Also catculute pw. leakage read. tance. ‘The data given ia: depth of hv, coiled mm, depth of 1. coil=36 mm, depth of duct behocen bv. and |.v.=16 mm, width of winding=0'12 ma length of mean turn=I'5 The number of turns in b.v. winding are 200. Solution, From Eqn. 7°80, leakage reactance referred to h.v. side, * mnxsoxanxiorx TS xp 0016+ 0088) 41g, HLV. winding current at full oad = 00x00 =50 A. 50x 1°41 vs Per unit leakage reactance =0'0953, id ELECLRICAL MACHINE DESIGN 7-48, Regulation. Fig. 4 17 (a) shows an approximate equivalent circuit of trans. former with parameters refered to primary side. 0} ram with laoging load, @ Fig. 7-77. Simplfied equivalent circut and phasor Qn no load di secondary terminal volage Vp'=Fp. ‘The drop in secondary terminal tage from no lor! to full load can be calculated by drawing the phasor’ diagram. 7°77 (b) shows the phasor diagram ata lagging power factor cos. From the phasor diagram : V p= A 04 Bap cos Ft TpXp sins OF Ky cos O— 1p Rp an OF Assuming that the angle @ between Fy and V,' is very small, we have : "y+ Ly Kp cos $+LpXp sin § or Vp~Vp'=IpRy cos $+-IpXp sin $e ed output Q and full load current Zp, i sing Vp ‘The p.v. regulation, for full load r Vom V9! _Jallp ons #4103 Vp Ve If the regulation is large and the phase shift between Vp and Vs! is not justifiel. For this Case the following relstionship should be us ete 20s bee ain $+HCs cos Omer sin iF, 7°82) Example 715, Estimate the per uuit requtation, at full load and O'S power factor ayying, for a 300 kVA, 50 Hz, 600/400 V, 3 phase, delta/atar, core type transformer. The data given is : LY, winding— ouside diameter= 0°36 m, inside diameter =0'29 m, arca of conduetor=6'4 mm’. LY. windis outside diameter =0°26 m, inside diameter=0'22 m, area of conductor =170 mm*, Length of coile=0°5 m, voltage per turn=SV, resistivity =0°21 Q/m/mm?, 400 0 oltage per phase y= =? . Solution, 1..V. voltage per phase Vimy = 231 V. =er cos dey sin 6 (781) eu L.V. turns per phase T= HLV. voltage per phase Vp=6600 V. Mean diameter of Lv. winding» 2284072 0.94 m, Length of mean turn of | v. winding=x 0°24=0°752 m. Resittance of Lv. winding rea 2221260752 _ 9 qo269 0. ‘TRANSFORMERS. ars Mean dian Length of m er of hiv. winding =(0". 6-0'20)/2 turn of hv. winding==m 0'325-=1 02 m. O21 x 826X 1 02 Resistance of hv, windin: cue. Resistance of transform referred to primary &, W100 36600 STA B26 +0 UOQGOLEIEA A) = 47. FLV. winding current per phase y= PU. resistance eri Ls 47. 1 PLU. resitaner erm 1A Mean diameter=('36+022)/2=0".9 m, Li ngth of mean turn Imes ™0°29=0 91 m. Width of Lv. winding 6.=(026—0 22)/2—('U2 m. Width of hiv. winding bo=(0'36—0'29)/2=0°035 m, Width of duct a=(0'29—-0'26)/2 =0015 m, = wUiL6, Leakage reactance of transformer referred to primary side ont 003540 a) Kom x 5X dmx 17 + (826) x TE (90153 173.0. 1x1-73, 05 SSI 0 0395 From Eqn. 7 81, per unit regulation emer cos $+6, sin d= ('0126 x 0'8-+0°0395 x 0-60 0338. 7-49. Mechanical forces, It ix well known that parallel conductors repel o attract each other according 10 whether the currents they carry are in. the apposite or in the same direction. Mechanical forars exist in transformers (even though they are. stationary) on account of the interaction of the current in the windings with the leakage firld surround ing them (windings). Te should bs usted that the two winclings carry currents. in the opposite direction, and pau. leakage reactance te= ‘The leakage ficld in a core type transformer is shown in Fig. 7 78(a). The kaye (@) Leakage fleld —(») Axial leakage field (c) Radial leakage fleld Fig. 7-78. Leakage fields and menachical forces. field can be resolved into its axial and radial components as shown in Fig. 7°78 (6) and ( The mechanical forces produced on azcount of int-raction of two components. of leakage flux with current are consequently in two :lifferent porpendicular directions The forces tn two different direct (i) radial forces and (ii) axial forces. 414 ELECTRICAL MACHINE DESIGN 1, Radial Forces. Fig. 7°78 (5) shows the interaction of axial component of leakage flux with the current careying windings. Applying Fleming’s lft hand rule it is found thatthe force acts in the ravlial direction. ‘The nature of radial force is such that it tries to. burst the outer winding (subjecting it to tensile force) and to crush the inner, winding (subjecting it to compresive foree). This is obvious, becauz: the primary and the secondary windings of a transform-r carry currents in the opp ite direction and thecefore produce a force of repulsion tending to pull the outer winding and to compress the inner winding. 2. Axial Forces Fig 7°78 (c) shows the interaction of radial component of leak- age flux with the current carrying windings. Applying Fleming's left hand rule, it is foun! that forces produced due to this interaction act in the axial direction. The forces acting on the transformsr winrlings inthe axitl direction wre compressive in nature. However, if there is an axial magnetic assymmotry between the two windings, a tensile axial force i » produced, NX ‘The mgnitude of force acting on a conductor is proportional to the product of current in it and the intensity of magnetic field duc'to currents in the neighbouring con. ductors, and since the latter is itself prop vrtional to the current, the forces are proportional to square of the current. The mechanical forces with normal load currents’are low compared to the strength of windings and hence, are not noticeable. However, when large transformers are subjected to short circuit con:litions at full voltage, the short circuit currents may reach 10 to 25 times full load value with forces increasing to 100 to 625 times undee. nornial conditions. Under fault conditions (such as short circuits or ocher severe faults) the mechanical forces may become so great as to damage the transformer permanently unless the winclings are solidly braced i.e, mehanically reinforced. 7-50, Calculation of mechanical forces, The mechanical forces produced in a transformer may be classified as : (i) radial forces, (6) axial forces, and (iii) unbalanced axial forces due to magnetic assymmetry. 7501, Radial forces. Consider a core type transformer with cylindrical windings as shown in Fig, 779, The winding has 7 turns and carries an instantaneous current j. "The instantaneous mmf-acting across the duct is £7. The flux, density in the duct is * pH =p iT/Le. ‘The flux density in the windings varies from 0 to Bs. ‘Therefore, the average value of flux densiy in the duct is Bs/2-or Bos=py 87/2Le. Radial force acting on a strip situated at a mean radius R within an angle d@ of AF turns flux density x current xlength of strip 2 Px Bo xix Rabe Dx PED xix Rat ery Ea Hence, total instantaneous radial fore acting on the coil 2n 2n Fen[ arm Gry E{ aom-t ory 22 é a 2 Te Rut 2eR=mean periphery.of coil= Lm Fem Be ye A788) ‘TRANSFORMERS 415, Now, short cireuit fault current is I'te where / is the rms value of full load current and Se is the per unit leakage reactance. (The resis- Prmary tance of the winding is neglected). On w winding fault conditions the instantaneous value of cutrent may become equal to twice the maximum vale of current under short circuit conditions. Fr under worst fault conditions f=2Y 2/0 Secondory Hence, maximum radial force under worst winging fault conditions is ” I Frmany-5t( 22-1 J HEN wary ae nind 2) lt Te (784) The ratio of maximum radial force under worst possible fault conditions to maximum radial force under normal conditions is : Fig. 7-79, Radial forces on windings Femeayt, 2iolV2I/60\Lm/ L, 2 Fence bal V Ulm] Le ty Itis clear from above, that the radial force is inversely proportional to square of the puu. reactance. Let consider a transformer with ==0'05. The maximum radial force acting in this transformer is 2/(0'05)*=800 times that maximum radial force under normal operation. ‘The average radial force under normal operating conditions is 2 Fam im. x) #21 dwt) an Int _! , am [ verso tate Im = cry 7"83) ‘The windings of a transformer must be designed to withstand radial forces under worst fault conditions, It is clear that circular coils should be preferred as they are the strongest to withstand radial forces. This is because the tendency of the radial forces is to make the coils circular and as they are already circular there is no deformation. However, if ree: taagular or square coils are used, they would be deformed by the radial forces.’ In shell type transformers, the coils are rectangular but these coils are mechanically braced for = considerable part of their length and hence are mechanically strong. 7582, Axial Forces, The winding currents interact with the radial component of __ leakage flux to produce axial compressive forces tending to squeeze the windings together at 416 . ELECTRIUAL mat nine sure the mide With symmetrical windings these forces are not noticeable even at short In shell ype transformers having, sandwich coils symmetrically arranged as in Fig 5 80, there isa force of ropubion Between the coils of each 3-4, 5-6, and 7.8. The force on coil 2 is opposite in direction to that acting on 3, likewise fot coils 4—5 and 6—7. Thus there is force acting on coils 1 and 8 only.. The magnitude of this force is computed as follows : Average value of flix density in the wincing, i iTy Fume Me ‘Ta=pumber of turns in the end half coil. Instantaneous axil force duc to interaction of magnetic ficld and the current ¢ in T's turns of end coil is : But Tam Las 2m inthe total numberof half cil vn 8( £0) Bh om) ~% Average value of this force is: Fa (7°87) 7503, Forces due to sssymmetry. The anaiis cared out il now thas been for syminetrical arrangements of windings. The axial length of windings is assumed equal with windings plecrd symmetrically with Fina F sion respect to each other. The mmf distribution in a ical arrangement is uniform and therefore the magnetic centre of the windidgs are coincl- dent. Fig 7'81 shows a winding arrangement where the two windings have an axial assymetry as a result cof which the magnetic centres are not coincident. cota Fite ‘The force, F, between the two windings can b> resolved into its radial and axial components. The fadial component of force is F cosa and the axial componentis P sing, The iatter may attain large values under fault conditions causing movement of wining fo he axial recon which, would fn- crease the axi stil farther. Therefore, in : ‘order to prevent axial movement of the windings, Fle 741, Fores ot magoetle Adequate bracing of the winding is necemary. nin teats of axa maymetey ote windings of tranforne. are provision of 8 regulation, man of certain go age re ing proces types of windings and non, , ‘These are ‘asunder t ‘TRANSFORMERS ~” 4 _ J One of the windings of the transformer, usually the h.v. jing, is provide. with tapings. These tapings include and exclude a part of the winding during service This causes mmaf uabalance between the windings in the axial dire 2. When calculating the axial length of helical windings, one turn is added to the number of turns due to helical wind of the turns. However, at.a greater width he of the turn, an empty space, free of a conductor material, is formed at the ends of helical (or cylindrical) windings as shown by hatched area This causes shortening of winding height Ly approximately 4/2. 3. ‘The end turns of the high voltage are reinforced to with- stand the effect of surge voltages. ‘Therefore, the end coils of the wine dings have lesser turns and hence the mmf distribution of the h.v. windings is not uniform along axial direction. Fie. 82, Halal The case of axial asymmetry caused by provision of tapings in the windings is show 7°83. Due to cutting out of a svetion of one of the windings tu provide voltage n tion, the axial length of the winding provided with tappings becomes smaller as comparec with the axial length of the other winding. The axial forces produced are on account of radial leakage field resulting from axial asymmetry of windings. Fig 7°83 (a) represent: worst conditions where the tap- pings are provided at one end of winding. The coils get further dis« -- placed due to assymmetry resulting 5 a farther inte ication of forces. the tappings are provided at both the ends, we have an arrange- | Je] ment shown in Fig. 7°83 (6). There : is thus ideally a lesser tendency to @ 6) © aggrevate the effect. Fig. 7°83 (c) shows the effcct of providing tap- pings at the centre of a winding. Fig. 7. Aria forces of aeymmetry. The force acting between coils of unequal lengths can be calculated by resolving the leakage flux into its axial and radial components. Fig. 7'84 (a) shows the cage of two win dings having unequal axial length, with each winding having an mmf AT. The arrange- ment is resolved as in (8) into two superposed mmf distributions (3) arld (ii) in accordance with the length wise unbalance k. ‘The first case (4) is dealt with as if it_ comprised balanced windings of equal axial lengths, and latter (si) in a corresponding way b t with different orientation and appropriate physical dimensions. The windings are assumed to be enclosed between two iron surfaces separated by a distance 2/ as shown in Fig 7 84 (c). er (RAD. | Ap. UNE et at @ ® (e) © Fig. 7-24. Resolution of mnaf of axially essymmetrical windings into two superposed mmfs, 418 where BLECERICAL MACHINE DESIGN Lead bpt-ls Average value of racial flux density Bary=4i0 aa Total instantaneous axial force acting on the coil / Fonte HOD (i TE Hy Hi Mae (788) Average ail force Fam HIM gr (7°89) Fig. 785. shows the diffirent cass of asymmetry in witr:ings along axial length on account of provision of tapings. Both radial and axial forces are_produerd. The values ofp.u reactance and the viluss of radial and axial forces pro tuced compared with those Produced with symmetrical windings are given. TH fo, poe all eM UL: a l of toot pu. reactance o1 os ou Relative p.u. stress: radial 100 50 a axis 0 8 9 TS1, Bracing of windings, Inward axial forces are taken by windings them. Inward radial forces are passed on to the formers, packing pieces and cores. Outward axial forces must b= withstood by end insulati Fig. 785. Axial assymmetry in cylindrical coils. A well constructed transformer should have suitable choice of conductor dimensions and interturn insulation, and the coils well m ted and bri easily arranged to give good insulation and at the Therefor xd with compressive stresses kept in view. The en! supports are not ime tim* good m‘cianical strength. , the nced for second quality (good mechanical strength) should be avoided as far as possible by having good magnetic symmetry and the unpreventive asymmetry caused by provision of tappings should be suitably distributed. Example 7-16 4 1000 kVA 6600/400 V, 50 Hs, 3 phase delta/-tar, core type, oil smmeroed transformer has 600 turns on the h.v. winding. The height of .inding is 0 Om ond the length of mean turn I'3m. Calculate the instantancous radial force owtbs hv. winding if Gahort circuit vccwre at the terminils of the Lv. winding with hv enargixgi ‘The leakage impedance ie 5 percent. Take the doubling effect multiplier as 1 8. Also caléulate the force at fall load. Solution. Full load current per phase on b.v. side = 100x 1000 _ ,.,.. ‘gx 6600 905 A Tastantaneous peak value of short circuit current mlx fT X (1/005) x 50'S 2580 A. From Eqn. 7°52, the total instantaneous radial force on the hiv. coil : ae Ge Eat SEX IO 135. ‘Fr 2 ry Te z (2580 x 500)* xoe 2°33 x 10" N. TRANSFORMERS a9 ot . Force at full load, Br= IO" (sy x 500j2% 18 = 066 N, This stows that the forces under short circuit conditions are considerably large as compared with forces at full load. Example 717, 4 575 kVA, 7500/4385 ¥, 80 Hzy single phase, core typ* transformer Ian the folidgina dita rian ‘hv. winding=27 mm, width uf Lv. winding=23 mm, width of duc! =15 mm, height of coile =0'35 m, length of mean turn =I'25 m, number of turn in ho. winding = 190, per unit impedance =0°086 ; doubling effect multiplicr~1 8. (a) Pind the inetantancous radial force on hv. winding under short circuit conditions the height of h.v. and Lv. winding is equal. (b) Find the instantancous arial foree 0% hv. winding wndrr short circuit conditions éf the he. winding ‘x 5 pricent aborter than the Le, winding at one end. Solation. Rms value of fall load curront==575 x 1000/7500— 6°7 A Peak current under short circuit conditions, m'8x VT x 1/0036 X76°7=5420 A. (a) From Eqn. 752, radial force Fr= Ae. ine a. 72 x (5420% 190)*x aE 2 80% 11 N. (i) From Eqn, 7°58 axial force Pool kGT! 4nx1077 1-25 z WO15-F0'027 0023) This shows that there is « very Linge axial force, even though one of the windings is only 5 percent shorter than the other at one end. 782, No lond current. The no load current [y consists of two components : (8) magnetizing current Jm (i) loss component Ji and its value is given by Jom (In?-+ J?) Thus the estimation of no load current Ip requires the calculation of its wo compo- nents Jm and It 7521, No lead current of single phase transformers, Let 1, ly=length of flux path through core and yoke respectively (=H, y=), le, dtys=mmf/metre for flux densities in core and yoke respectively. . Total magnetizing mmf AT,=2 ate le+2 aly ly+mmnf required for joints ...(7°90) ‘The values of ate and afy are taken from the B—H curves for transformer stecl (given in chapter 4). The joints in a magnetic circuit may be taken as short air gaps in parallel with iron paths. The calculation of total mmf is b cd up.on the maximum value of flux density, Tmt 2 (aby tb) %.0.05 » (5420 x 190)8X =3'2X 10° N. 2. Runs value of magnetising curs nt ImeeATy/(/2 Tp) wa(791) But the magnetising is not sinusvidal and therefore the peak factor Kp should be used in place of / 2. In AT o|(Kox Tp) wal P9") Let the iron losses be Ps, the loss component LaDy ion \ 420 ELECTRICAL MACHINE Dhxi7 The iron losses are calculated by finding the weight of cores and yokes. The loss per kg i taken from relevant curves given in chapter 4, 7522. No load current of three phase transformers, Total magnetising mmf required for the transformer =3 ate Le+2 aty ly-+mmf required for joints. ‘Total m:gnetising mmf required per phase AT = Bite 2ataly ttm required for joints Magnetising current per j hase Im=ATo/(F Ty) or Im=ATo|(Kpt Tp) «..(7 98 ‘The values of ate and aty are takeri from relevant graphs given in chapter 4. Let Ps be the total ircn loss for the three phases T= PY 3p (794 Te isnot usual to calculate the no load current in transformers as in a normally designed transformer, the no lvad current would be below 5 percent and a considerabl. variation can be made in its value without effecting the transformer perfurmance, 751-3, Magnetizing volt-ampere. We have Ey=4'44 f Ty Bu Ai Mognetizing volt ampere (VA)m=Eplm=(4'44 f Tp Bn Ae) ( & - ) wt 44 f Buds AT, 2 Now, ATy=magnetizing mmf per metre Xlength of path in i-on=atale Aan 4 [Be “ ain le Weight of iron=As li 7 8X 108, if the density of iron is assunicd as 7°8X 10% kg/m* ‘Magnetizing volt-ampere/kg 3s 10°4 fBmate x10°*, 2 xTBx 10° Now a curve can be plotted betwoen Bu and magnetizing volt ampere/kg from Js—-H curve of the material (valuc of atm for different valucs of Bm will be known fom this B—H curve) Manufactures supply the magnetizing VA/kg—Be characteristics and from it the magnetizing current can be known. Magnetizing current ‘n_magnetizing VA/kgX weight of core . In —Tumber of phases X voltage/phase +796) Example 718, A single phase, £001, 50 He, tranyformer is built from slampinge having a relative permeability of 1000. ‘Tha lengih ofthe fur path ie 25 m, the area uj eros acclion of the core is 25% 10-* m¥ and the primary winding hae 800 turne, Ealimate the mazimum flus and no load current of the transformer. The iron loss at the working flua ee ee Win Len weighe ed sto poe Stacking factor ia 09, fe Solution, Net iron arca A¢m0'9X2'5 x 10°tm2°25 X 108 mt We have, By=4'44fOm Tym4'44 f Bu At Tp. , - 400 oH “ Bw FAG 50X25 X10 2X 800 1°C Whim’. Ss Flux in the core= By dem] X2'25 x 10225 x 107 Wb, TRANSFORMERS aun ai k Mnagetizing mmf AT,=relucta flux = ——-XOma= - ageizing srluctincex fax = = xan 25x1 6 1000 x dre x 19-77 1280 4 . AT, 1980 ‘M agnetising current I= we p= 1°75 A ® V2 “TE 000 Volume of core=2 25 x 107? x 2.5=5°625 x 107%, Weight of core=5 625 x 10-*X7°8X 10°=43'H kg aid iron loss=2 6 mw =144 W. ‘Loss component of no load current I= e+ No loa! current Jo VT75"+0785"==1°77 A. Example 7-19 Calculate the active and reuctiv: components of no load current of a 100 V, 50 He, single phase transformer having the following particulars Core of transformer steel; Stacking factor=0 9; density ~7'8 x 10 kg/m? ; length of mean fluz path 2°2 m ; groas iron section 10% 10? m* ; primary turns 200; jointe equivalent to 0°2 mm air gap. Use the following data : an ~ ; : : , we |» [| | = |» |» in | 130 | 2.0 420 | 660, | 1300 Tron loss ' : corte | os 13 19 | 24 | 2 Solution, Gross iron arca=10x (0- m*, Net iron area==9X 10 m?. 400 . 3 FTES SOR TOG =P 02 107? Wh. Flux density Bam $2=9'02x 1074/2 x 10-¥=1 W/m. Corresponding to Bu=1'0 Wb/m!, mmf/metre=210 A, loss per kg=I°3 W. Mnf for iron path=210 x 2°2—=462 A. Mat for joints=800,000 Bly=800,000% 1 x (02 = 160 A. Total magnetising mmf, AT,=462+160—622 A. Reactive (magnetising) current, Jn== Te =22 A. Fluxincore a=, Volume of core =2'2 x 0°009=0°0198 m*. Weight of core =7'8x 10*X0'0198=155 kg. Total iron los= 155 x 1'3=201'5 W. Lous (active) component of noload current lim "Qh 40'S A. s+ No load current=4/ Fat FIt=d 22°40 5'=2°26 A, Example 7'20. 4 6600V, 60 He single phase transformer has a core of sheet steel. ‘The net iron cross ectional area ie SECIS oP the maan length 828 es Gad thareare aun ELECTRICAL MACHINE DESIGN Jour lap joints. Each lap joint tukew 1/4 times ae much seuciive mmf us ia required pec metre ‘of core If By=1T Wo'm?, determine (a) the number of turns on the 6600 F winding and (by the no load current. Aasume un amplitude factor of 1°52 and that for given fluc density, min] prv metve= 282 Alm ; apecific Lua= 1°76 W/kg. Spreifie yravity of plaies=7°5, Solution (a) Number of urns . () Mint required Mnf r. quired for jointomed » $4 292282 A. “Total magnetizing mmf ATy=517-+232= 749 A. Mugnotizing current Ima -¢42¢— =? 3 Ent TSIRV2 ‘as peak factor Kya=amplitude factor x ¥2) Weight of core=2'23 x 22 6: 109X7 5x 10378 kg “Total iron loss Pr=1°76 x 378=665 W. Loss comp aunt fa Seo a. No load current Lgme@ (0318) (0 1}*=m0'333 A. Example 7-21, A 15000 bVA, 33/66 kV, 3 phase elar/delta, core type transformer as the following data : Net iron area of each linb=1 5x 10° mt, not area of yoke=1'8x 10-* m?, mean length Of fiuz path in each} mb=2'3, mean length of luz path in each yoke== 6, number of turns in winding = 460. GCaleulate the wo load current. ee the data given in czample 7.19 for mmf per metre and loos per ig. elution, H.V. winding voltage per phase=33300//3=19100 V. 19100 . Flux Sam Fax G0 KET TO 1! We Flux density in the limbs, Be (limb)= 2734 1:27 Wojia?. Corresponding to this flux density, atem560 A/m ; p= 2°25 Wikg. Flux density in the yokes, Bp (yoke) £191 Spe 106 Wh/m?. O18 For this fux density, ofy=260 A/m ; po=1"4 W/kg. ‘Total mmf for three limbs =3X560X2'3=3860 A. ‘Total mmf for two yokes ==2X 260 x 116832 A, Total mmf for limbs and yokes =3860+832—=4692 A. ‘Magnetizivig mmf per phase 47 =4692/3= 1564 A. cs Magnetizing current per phase Zam 156 -/(/2x 450)=2'46 A. Volume of 3 limbs =3X2'3x0'15=1 035 m!. Weight of 3 limbs =1-085X7°8 x 103808 10? kg. Volume 0 two yokesm2X 1'6X0"18—0°576 m?, ‘Weight of two yokes==0'576 x7" x 10'=4°49 x 10° kg, ‘Loss in limbs =80Bx 10°X.1:25= 182 x 10 W. TRANSFORMERS as Loss in yokes 4°49 x 108 14563 x 108 W. Total iron loss 18°2 X 108+ 63 x 10% == 2 <1 Ww, Toss per phase =24°5 x 04/3=8'16x 168 W. "Loss component per phase [1=8'16 x 103/19100=0'427 A. No load current [5=¥240°+0427=2'5 A. 7/53, Change of parameters with change of frequency, 1, Effect on Losses, The specific iron loss is given ky pi ~KifBm*+Kif*Bnt W/kg. From this relationship it is evident that change of either f or Bm or both, will, in eral, result in change of iron losses. As the transformer is designed on the basis of lefinite heat dissipation, the J2R loss should be re-adjusted if the tbte! losses are to remain the same i.e, if the core loss under changed conditions is greater than that in the original design the Z2R loss will have to be made smaller, and vice verea. With the new J2X loss, and effective resistance of winding, the new current rating is easily obtained. Let us examine the effect of change of frequency on iron losses if the voltage remain the same. We have voltage Badd] BoA and eddy current loss Po=K, f*Bat. ‘Therefore as long as H remains constant product (fm) remains constant and theres fore, eddy current losses remain constant even though the frequency is changed. We have hysteresis loss Pa=KafBn Now if applied voltage is kept constant product /Bm rcmains constant. Let fBa=K. Hysteresis oss KY JB) Bae KA B= Kak Am Ak . From above it is clear that the hysteresis losses will decrease with increcse in frequency if the voltage is kept constant. ‘Therefore, the total iron losses decrease if the frequency is increased and the applied voltage is kept constant. Hence with increased frequency, we can afford to have more I'R loss and thus for the same Loss (i.e. the same temperature rise) the rating of the transformer can be increased. ‘This fact is illustrated by example 7 23. =. Effect on voltage. If :he flux density is als» changed, the new voltage rating is determined from the emf equation. For the new voltage total iron loss and the magnetising currint and the no load current can be calculated. 3, Effect on leakage reactance frequency will not have much effect on leakage i will increase linearly with frequency. Due to skin effect, the effective resistance increases with increase in frequency. This effect is not normally important with small changes in frequency. Example 7-22, An 11 kV, 25 Hs transformer has I°R, hysteresis and eddy current lossca 1'6, 0°6 and 0'd per cent’ of the output. What will be the peroentage - loasea if the transformer is connected (0 22 kV, 50 Hs eupply aseuming the full load current to remain the come? Solution, Subscript | refers to case with 11 kV and 25 Hz, Subscript 2 refers to case with 22 kV and 50 Hz. (a) AR lors. As the fullload current is same in both the cases, the copper low remains the same. PR losses = Psy Pev='6 per cent of output with kV. bed ELECTRICAL MACHINE DESIGN The transformer is assumed to be single phase. B= 44d fm, Land E or or” Se Bay= Boy (0) Hysteresis loss. Pa=KaDnt {or peat or Pxy=0'6x 32 =12% of output with 1kV. | * (@) Bédy current lon PcKeBatf* or is aoe ea a or Popa 0-4 x SE 16% of output with 11 KV. ‘The output with 22 kV is double that with 11 kV as the curren’ is the same in both the cases. Therefore, PR, hysteresis and eddy current losses are respectively 0°8%, 0°6% and 0°8% of out- put with 22 kV, Example 723. 4 40 Hz transformer ia to be used on a 50 Hz system Assuming the Bteinmiets's co-efficient as I°6 und lossrs at lower frequency 12%, 07% and 0 5% for I*R, Aysereaio and eddy currents respectioely. Find a) Lute om 60 Hs for the asme supply volge ‘and current (b) ouput at 50 Hz for the eame total losses ps on 40 Hs, Solution, (a) The voltage and current at both 40 Hz and 50 Hz are the same and therefore the output in both the cases is the same. Subscript | refers to 40 Hz aud subscript 2refers to 50 Hz. ‘The current at both 40 Hz and 50 Hz is the same and therefore the J*8 loss is same in both the cases. Pix’ By_ 4°44 fxm? We have, 5 af “ Oag= On, X £. fF Oy, =0' 80m oF Bey =08 Buy. . = My Pha Kang fz _ (08 Bo)! Ye Now, hysteresis loss Pa==KuBm "YY or pit pet lO a he 61%. "a= 12%. and as Ey, or Page Pn,x0'8*X 3 =0°875 Pry =0°875 x0'7. ed current loss Poe KBagit or Fitmpneagt moray ($0) ay ‘ P= Pa=0'3%. (0) If the rating of the transformer is changed, eddy current and hysteresis loses remain the same while there is a change in 122 loss. Total loss in Ist caseml'2+0°7-+0'5 = 24% Total loss in 2nd casem 1'2+0°61 +0°5=2'31) \NEFORMERS: . 42 In order that the loss should be equal in both the cases, the *R loss has to be increas: 2°4— 1°31=80°09% in the 2nd case. - PB loss in 2nd. case 1°2-+0°09—1°29' PB lons varies as the square of the output eal Seip aa as y output in second casemoutput in Ist casexfPajPamoutput in st case x29) 2 _ + 1°038% output in Ist case. ‘Thus an increase in frequency results in increase in rating. | 754, Temperature rise of transformers, The problem of temperature rise and joling of transformers is casentially the same us that of rotating machinery. Similar to the fer, the lowes developed in the transformer cores and windings during conversion are werted into thermal energy and cause heating of corresponding transformer parts. Frum. f source the heat is directed, clue to thermal gradients, to the places where it may b: trans- ‘fred to a cooling medium i. to air, or water, depending upon the method of transform=r oling. Heat dissipation occurs in the same way as in clecirical machines, i.e. by way of Wiation and convection. The path of heat flow is : (8) From the internal most heated spots of a given part (of core or winaing) to their uuter surfaces in contact with the oi }) From the outer surface of a given transformer part to the oil that cools it. (iii) From the oil to th walls of a cooler, for instance, of the tank. (io) From the walls of the cooler to the cooling medium—xir or water. In ection (i) the heat is transferred by conduction In sections (ii) and ( tansferred by convection of the oil | In section (ie), the heat is t the heat ix ation. ipated by both convection and r. 754-1, Transformer oil cooling medium, Tests have shown that an yerage working temperature of the oil =50 to 60°C and oil viscosity corresponuing to this emperature, the specific heat dissipation due to convection cf éil is Deane 4030/4" W/m" there Om temperature difference of the surface relative to oil, °C Hosheight of dissipating surface, m. If we assume an average 6=20°C and H=0°5 to 1 m, Acens==80 to 100 W/mt—°C. The corresponding figure for convection “ue to air is 8W/m?—"C. Thus the convec- ion due to oil is 10 times and above that with air. This constitutes a major valuable ro rerty of oil as a cooling medium. 7542, Temperature rise in plain walled tanks, The walls of tank dissipate t by both radiation and convection. It has been found expcrimentally that a plain tank face dissipates 6 and 65 W/m'—°C by radiation and convection respectively (lor a tem- rature rise of nearly 40°C above an ambient temperature of 20°C. Thus the total loss ipation is 12°5 W/m*—"C, . : . : total loss Pct Pe . s+ Temperature rise =, - a -heat dissipation Xsurtace™™ 12°5 Se (797, where Smheat dissipating surface of tank. The surface, to be considered in applying he above formula, is the total area of the vertical sides plus one half area of the cover, unless he oil isin contact with the cover in which case whole area ofthe lid should be taken. “he area of bottom of the tank should be neglected as it has very little cooling effect. 426 FLECTRICAL MACHINE DESIGN For transformers of low output, the plain walled tant -ge wre enough to stecommodate the transformer and oil have sufficient surface to keep t —smperature rise within limits, But for transformers of Large output, the plain walled tar re not sufficient to. dissipate losses. ‘This is because volume and hence losses increase. be of linear dimensions while the dissipating surface increases as the square of linear « — nsions. ‘Thus an increase in rating results in an increase in: loss to be dissipated pert area givinga higher tem- perature rise. Modern oil immersed power transformers with natural oil cooling and a plain tank may be produced for outputs not exc ding 20—30 kVA. ‘Transformers rated for larger outputs, must be provided with means to improve the conditions nf leat dissipation. ‘This may be done by providing corrugations, tubing or radiators where feasible. 7°55. Design of tank with tubes. If (1c temperature rise as calculated with plai tank excecds the specified limits, it can be brought down by provision of tubes. The pro- vision of tubes increases the dissip iting area but the increase int dissipation of heat. is not pro- portion: to area because tube screen some of thu. tank surface preventing radiation from there. So there is no change in surface as far as dissipation of heat due to radiation is concerned. But the increase in dissipation of heat is more than what is justified by the increase murface area. ‘The circulation of oil is improved due to more effective heads of pressure produced by columns of oil in tubes. An addition of about 35 per cent should be made tu tube area in order to take into accor ~* this improvement in dissipation of loss by convection. Tat the disipatieg surtvee of the tak be St Tt will dissipate (6 #0 5) Si WiC Loss dissipated by tubes by convection==1°35 X6 5 1 and tubes Lot the Sit 8 8 28 Si (125-48 6 7) WC 288i (12) ‘Total loss dissipated by tank Total ares of tank walls and tubes Sr (SERED seg a Wine" os ctissipated = Pathe SUES Fare Temper. ture rise with tubes @ Total area cf mabesset (ME ia Let fe and de be the length aud Areaof each tube dil, -1255:) ameter of each tube respectively. C ex tf Pete igs Hence, number of tubes me= picegg-( —g ss) ene('100) ‘The arva of the tubes can be found out by using the above expression. ‘The diameter of tubes, normally used, is Sti mm and ney are spaced at 75 mm Elliptical tubes with ‘Pressed radiators are increasingly beng used asthey give a greater dissipating surface for smaller volume of oil. The inner dimensions of the trarsformer tank are fixed by the active dimensions of the transformer and clearances betw cen windings and grounded parts of transformer. Width of tank Wi=2D+4+0-+26 (for three phase} =D+De+26 (for single phase) wire D=distance between adjacent limbs, D,=external diameter of h.v. winding and ‘TRANSFORMERS beck utnce between hw w Length of tank Le, where (clearance on each yide between the winding atid tank along ¢ Peo . Fi, 746. Tank dimennons. Height of transformer tank H=H +h ne width, 427 where H=height of transformer frame and A=clearance (height) between the assembled transformer and the tank, This includ es the clearance at the b, assembled transformer space fur terminals and tap changing gear clearances by 1 and h are given below : °, oi height above the ‘Typical values of Voltage Rating Clearance Ww KA i A 11 kV or leas | less than 1000 4 | 50 450 1600-3000 1 | % 40 about 11 kV Jess than 1000 1s | 100 550 and upto 33 kV 1000-5000 as | 125 550 7-56. Air blast cooling, At present time the radiators are forced cooled by small fans mounted on each radiator. _Curapared with natural ecoling, air blast cooling of a tank increases the heat distipation by 50 to 6U per cent or ‘=loss dissipated W/m*—"C by both convection and radiation with air blast. = (1'5 to 1'6) x 12520 W/m'—"G. 2B ELECTRICAL MACHINE DESIGN Transformers upto a capacity of !9 MVA have a cooling radiator system with natural coling. For 10 MVA upwards air blast cooling of radiator 1s used. 787. Forced oil circulation. The velocity with which natural_oil circulation takes lace is very low, of the order of a few mm/s. Investigations of te problem have aown that with an increase in velocity of vil circulation by m times, the transformer output, > the same winding temperature rise, increases (m)!/* times. If for example m=3, the ‘ansformer output rises by about 30 per cent. An excessive increase of oil circulating rate ‘unsuitable because ths involves large energy losses in the purging unit. To cool the il, itis circulated ‘trough a special oil rooler. In an oil cooler with natural air cooling the flow rate of the circulating oil is of the irder of 12 litre per minute per kW of losses, When the cooler is air blast cooled, the transformer output increases roughly to the ame extent as when the tank is air blast covled. Ln transformers with water cooling of the circulating oil, the pipe coils and tube olers are employed working on the counterflow principle. Cooler surfice per 1 kW of osses ranges from U'18 to U'25 m', ‘The flow rate of the circulating oil per kW of losses, quals approximately 6 to 8 litre per minute. The water flow rate is about 1.5 litre per Rinute. ‘The temperature difference betveen the incoming and outgoing water is usually aken to be nearly 10°C. . Example 724, 4 250 4A, 600/400 V,3 phase core type tranajormer has a totat 080 of 4800 W at full load. ‘The transformer tank ia 125m in height and 1 mX0'5 m in lan. Design a ewitable acheme for tubes sf the average temperature rise is to be limited to 35°C. Uhe diameter of tubes is 50 mm and are spaced 75mm from each other. The averaye height J tubes ie 1°05 m. Specific heat dissipation due 1o radiation and convection is reapectively 6 and O'5 “C. Assume that convection ie improved by 35 per cent due to provision of tubex 1-071 1°25=3'75 mt Wh Solution. Arca of plane tank S Let the tube urea be 281. Total dissipating surface=(1+2) S=3°75 (1+2) 48 365 THI +2)X35 ~ THe W/m'—*C Specific loss dissipation = 1 82 Wire From Eqn. 798 loss dissipated=—7— W/m*~°C. or 2=2°73 ~ 1 ke J. Area of tubes=2°73 x 3°75= 10°23 m*. Wall area of each tube= mde l= 0°05 x 1°05=0°165 m*. Total number of tubes to be provided==10'23/0 165=62. ‘The tubes are spaced 75mm apart. Therefo.e in 1 m along the width of tank, we can accommodate 12 tubes leaving 90 mm on each side. in O'S m along the depth of tank we can accommodate 5 tubes with 100 mm space on each side. ‘The total tubes provided in the first row along width and depth are 2X12+2x5=34. The balance 62—34=028 tubes can be provided in second row at the back. TRANSFORMERS 42 iL tubes can be provided ina staggered fashion in each of the two long sides am 4 in cach of the wo shore sides. se . “e ‘ ongees® ¥d6000000000 900000000000: 20090: P| pop Oo000 oOO0000 CooO000 ooo°o eco0o0oO0CCOoO0oocCon O9N000000000 Fig. 787. Arrangement of cooling. tibes s+ Total tubes provided =? x 12+2 x 1142 x5 42% 4=04 Fig. 7°87 shows the arrangement for tubes. Example 725 4 1000 kVA, 6600!440 V, 50 Iz, 3phixe, dellufatar, core type, oi immersed natural eooled (ON) tranaformer The design data of the transformer is’: distance between contica of auljacent limba=0'47 m, outer diameter of high voltag winding =0'44 m, height of frame=I°24m Core loss 37 FW and FPR loss=10'5 kW. Design a suitable tank for the transformer. The averaye temprrature rise of oil shoulc not ercred 35°C. The apecific heat dissipation from the tunk walls is 6W/m?—C! and 6°5 W/m?—C du 10 radiation and convection reapectively. Ansume that the convection ia improved by 35% dui to convection. Solution, The clearance between windings and tank is assumed as 70 mm anc 90 mri on each side along width and length respectively. Width of tank Wi=2D+D.+26=2x0°47 +0°44-+2X0'07=1'5L m. Length of tank Zi=D+-+2lm0°44-+2 x 0°09=0°62 m. Allowing 50 mm for the base and 300 mm for oil above the frame, Height of oil level from the bottom of tank=1'24+0°05+0°3 ot 1'6 n. A further height of 300 mm is necessary for the leads ctc. <. Hoight’of tank Hi=t'6+0°3=1°9 m. Dissipating surface of plain tank S=2(152 +0°62)X 1'9=8'132 mt. Tet the area of tubes be = 5. ‘Total dissipating area=(1-+2) S:=8°32 (I-+2) mt 430 ELECTRICAL MACHINE DESION 1254802 Ite 4S KAY. Tie temperature rise is to be kept below 35° 52x 102 a Menew, end'sh 2. Area of tubes . 4 Specific loys slissspation dee W/mt—"C. ‘Total bye Telit Spwesfic Le slissipation= ately [4m and the diameter of each tube ihe sete feng of anise is appr is 0 nun, * n s+ Dissipating arcs of each tube=m de lezen <0°05 x 1°4=20°22 mt, ¢ 34.56 1 Hones, suimber uf tubes required Nes Sw i ber uf tubes requir var 733 ‘Lhe tubes ate spaced 75 ram apart. The arrangement of tubes is : Along length : 4 rows on cach side with 18, 17, 16 and 15 tubes Along width + 4:-rows on each side with 7, 6, 9 and 4 tubes Total number of tubes used =2 (18+ 17+ 16+ 15)+2 (7 +6-+5-+4)= 156. 7°58. Thermal rating. The rating ofa transformer is bared exclusively on ther- th fimit tion beng imposed by the maximum working temperature of the winding which will afford a reasonable life to the insulation, ‘The critical component of most trans- former insulativa is cellulose, which deterinrates. physically or mechanically at a rate deter- mined by moisture content, electric stress and oil purity. The most important factor is temperature. Insulating materials, immersed in oil maintaine| continuously at a tempera- ture 75°C may have a life of 50 years, but low limit of temperature would be uneconomical from the rating view point Tf the insulation is kept in oil maintained at a temperature of 10°C continunusty, the lite may be less than 5 years. Thus this would require transformer replacement after a very short time. Therefore, the transformer should not bs operated at ahigh temperature (for the sake of getting ‘higher output) otherwise its life will be very short. The reference ambient temperatures assumed are : mal ba (a) maximum ambient alr cemperature—45°C_ (8) maximum daily average air temperature —35°C (0) miximum yearly average air temperature~-30°C. Operation of a transformer at its rated kVA provides normal life expectancy if the temperature of the cooling air docs not, at any time, exceed the reference ambient tempera- ture, This results im copperin the winding atiaining a temperature of 90° to 106°C (ie. 55°C above maximum reference ambient temperature of 35°C daily average and 45°C maximum! and an assumed hot spat temporature in the windings of about 105°C. A rough and ing the deterioration of insulating materials at high temperatures which \ceeptance is that the rate of deterrioation of class A insulation in oil roughly doubles for every 8°C rise in operating temperature. From this its apparent that operation of a transformr for any given period in a cooling air temperature in excess of reference ambient temperatures such as to produc» rise in the temperature of insulation ‘of 8°C above normal is equivalent to its operation under standard ambient conditions for roughly double tlt: period. This would, of course, mean a corresponding reduction in the Tife expectancy of transformer. ‘On the other hand, it is possible to operate a transformer in cooling air tewy2retures exceeding reference ambient temperatures whilst still maintaining its normal life een ctancy by suitably reducing its load from its rated kVA, As a very rough guide, #* may be stated RANSPORMERS 431 that for a transformer covered by IST. specification 20261962, a reduction in load from its rated AWA of approximately 2 prrcent as niecossstv for this purp ome he each ISC by which the actual ¢ oling temperature evcceds the reference ambient temperiturr This tule docs not hold good ii the cooling air wimperature exces the reference ambient temperature by more than 10° If the sanuse is less than the reforener ambient temperature it is possible to op at toad above ate rated RVA by about per cent por dogree centigrade by which cooling air teaperaiur is Jess thats ther fers ev tir ambient temperature This holds good at th costing sir temperature drs st if ence ambient veinperature by m1 an 10°C from the refer. Whe new r the trassformer has a temacratite rye bese tia its speefied limit, i. may be subj eted to overlox! on thermal con sderations. “The time for which soel ay averload can be sustained dopends obviously on the initia tempe ‘Lom che time gonstant, The greater the wright of the transformer, the greater will be its thermal capacity an | time constant. Thus transformer with Lirge weight hasa smaller temperature rise for short time overloud:. Te is p sible te eonstonet a transformer of email ver jght to confirm with the standard ratings est or temperatu ere of windings and oil by proviring, ample tank cooling surface and adequate ducty in the windings But the performance of sacha low weight transformer will be unsatisfactory on short time overloads owing to its small time constant However, it is incorrect te compare two ttansformers in this respect solely on the das of weights without a know ledge of their cooling The ability af a transformer to withe stat d «ee.toads depends upon the eficieney of the cooling system as well as an weight. The overload that can be imposed upon a transformer depends upon the Joad copper oss Pe to iron Joss Ps. fall Let ns comprre ive transformers, one with Pe!Pi=] and other with Pf Pem2, At 200 per cont overload, the first transformer has a loss ratio of 41 TT ‘otal | total lo: ath and the second transformer has it loss ratio of otal loss at overload _ (2) x2+1 loss at full load ~ 241 fore overload losses in (wo cases are 2°5 : 3 ; $0 that a transformer with a greater Pr|Pe ratio is less capable of sustaining overloads. 759, Momentary overloads. ‘Transformers in service must be capable of wi standing short-circuits at normal line voltage without injury. The duration of short circuits according (0 IS ; 2026-1962 is : Percent impedance 5 6 7 and above | ~ “PF ~ i ration of short circuit i Duration of shot a 3 : 5 ‘The calculated winding temperature must not exered 25 °C starting from an init value of 90°C for water co ling and 105°C for air cooling. It is assumed that"the heat gene, rated is stored in the copper and there is no dissipation as the interval is very small. 432 ELECTRICAL “IACHINE DESIGN ‘mperstoe tne Oma 20a where t=time, s; T= 0 +235° 8, =initial temperature, °C Keeddy current Juss ratio at 75°C a= 0 0025 x Inss in W/kg ot 8 = 1-9 389", x 107% ant S=current density, A/mnt, = AT 101) 7-60, Heating time constant of transformers. The heating time constant is Tra=GhISA (see Chapter ig {hts means that te heating time constant T's is directly proportional to weight @ and specific heat A while at iy inversely proportional to specific hest dissipation A. The value of heating tume constant lies between one to tive hours. Natural cooling and high voltage insul- ation, which try to give a reduerd valne of specific heat di-sipation as produce longer time ceustants., A transformer with large weight has a longer time constant and is thus eapable of withstanding momentary overload. better thin a light weight transformer (as in a trans- former with a longer time constant, st takes longer fime for the temperature to rise). A transformer is not a homogeneous body and therefore the aboue rclationship ‘s not sirictly applicable to it. Asa transformer consists essentially of a core» a winding and oil, a.1 having different weights, specific heats etc. we have to deal with the heating time constants of core ,, the winding Ty and the oil To. Example 7-26 shows how greatly the heating time constants differ for the different Parts of a transfi rmer. Example 726. 4 3 phase 5000 kVA, 33 kV, 60 Hz, oil immeraed tranaformer has the Jollowing data : Weight : core=5200 kg, copper= 1200 kg, oit==5500 kg Core losser= 18 ¢W, copper losacem57 kW. The specific heats of core sicel, copper and oil are respectively 480, 390 and 1670 J/kg Assuming that (le temperature rie of core above oil 820°C of the winding above oit 20°C and the temmrature rise of oil Oo=40"C. Fil the healing time constants for core, winding and o Comment upon the resulta Solution. Heating time constant Ta=@h/SA but Q=58 or ShmQ/’—c Tam OBO @ stands for loss in the above relationship. Heating time constant for! p= 520 480220 T5000 1200%390%20. 37000 . 5500 x 1670 <0 Oil's Te= “Te000457000 Thus the heating time constant for oil is 30 times that of windings. ‘This makes it legitimate to assume that during rapid changes in load, we may without any #i,~ihewut error, neglect the oil temperature variation as comparcd with that in the windings.” Tuere- fore, the oil temperature is Ko indicative of the winding temperafure during r Diu changes Core: Te =2710 s=46'3 minutes. Winding : Ta= 164 s=2°74 minutes. = 4900 s==1 “7 minutes. TRANSFORMERS 433 of in load. Hence, in aildition toa temperature indicator immersed in oil another tem Perature indicator must be used to monitor the temp-raturc of the windings. DESIGN PROBLEMS Problem prsigna 25 KA, 1000/4351", 50 He, 3 il | ma 26 HA, 31. 50 Hz, 3 phase, delta tater, core type, oil immersed natural contd distribution tansforruer “The transformer x poorided smith toppings sé 24 £5% om the hy winding, Masimum temperature sige not to crend 45°C with mean temperature rise of ail 35°C. Solution, Core Design. The value of K ix taken from Table 2°2 K=0 45 for three phare care type distri Voltage per turn Kr=K¥ Q7045 V25=2°25 V. E 225 . Ch aang Fax =0010135 Wh, Hot rolled silicon steel grade 92 is used he value of flux densit vo wi steel grade 92 is used. ‘The value of flux density Bm is assumed us Net iran ar n transformers ss Flux in the core Ow= 0010135 5 Aco SOE 2 0:01035 m= Using a cracitorm core, Ar=0°S6 a 0135 x 103 mm* 134°5 mm Diameter of circumse Reference Figs. 7 a0 85 d The laminations arr punched from 750 mi wide plates « dimensions are a=114 mm cul h=73 mm Window Dimensions, The window space factor for a small Eu=8/(30+KV). . Ku==8/(30471 The current density in the windings is taken as 2°3 A/mint, ing circle dl Sar (2), widths of laminations and 85. 135 R= 1148 mm, b=053 d=0 53% 1358716 mm. thie nearest standard i transformer is 1195, The value acsumed iy Ke 0°18 Output of transformer. Q=3'33 f Bm Rue 8 Aw Aix 10 25—=3°53 x 50% 1 x0 18% 23 x 1OPX Ae X 0010135 10° or window area 4e=0°0358 m*=35'8X 10" mm, Taking the ratio height to width of window as 25, FoeX Wo=35'8 108 of 2°5 Waka 8X 10" S. Width of window We=120 mm and height of window=300 mm. Area of window provided A~w=300 x 120=36 x 10% mm? ==0 036 mt. Distance between adjacent core centres D= Ww-+d= 120+ 135=255 mm. Yoke Design. The area of yoke is taken as 1°Z times that of limb <. Flux density in yoke=1/1'2=0'833 Wb/m*. Net area of yoke=1'2 x 10'135 x 108=12°16x 10" mm*. Gross area of yoke==12°16 X 103/0°9«=13'5 x 108 mm’. Taking the section of the yoke as rectangular. . Depth of yoke Dy=a=114 mm Height of yoke Hyae13'5 x 10%/1 1408114 mm Overall Dimensions of Frame. Reference Fig. 7°70" ‘Height of frame H=Hw+2Hy=300-+2 x 114526 mm. 434 ELECTRICAL MACHINE DESIGN Width of frame W=2D-+a=2%255+114—624 mm. Depth of frame My=a=114 mm. LY. Windiny Secondary line voltage =433 V ; Connection=star. Secondary phave voltage Vs=433/9/3=250 V. Number of tus per phase 7'=V./Hi=-50/225=I11. Secondary phase current [= Re as 3A. A current density of 23 A/mm? is used. Area of seconlary conductor or=353/2'3—14 48 mm? From Table 17°] (IS : 16971962), using a bare conductor of 7°7%2°2 mm, Area of bare conductor a. 44 mm*. yy in secondary winding 8=33'4/149=2 23 A/ent, ‘The conductors are paper coverrd. The increase in dime covering is 0°5 mm. Dimensi of insulated conductor==7°5%2°7 mm}, Using three layers for the winding. Helical winding is used. ‘Therefore space has to be provided for (37-+1)=38 turns along the axial depth. Axial depth of Lv. winding Lea=38 X axial depth of conductor=38 X7°5=285 mm. The height of window is 300mm. This leaves a clearance of (300—285)/2==7°5 on cach side of the winding. (The minimum clearance should be 6 mm for windings having voltages below 500 V.) Using 0°5 mr pressboard cylinders between layers. Radial depth of Lv. winding, dyesnumber of layers X radial depth of conductor-+ insulation between layers 3X27 42X0'5=9'1 mm Fig. 7 88 shows « cross-section through L.v coil. Current de mason account of paper 5 mm Press board s ‘025mm paper tape, TRANSFORMERS: 435 Diameter of circumscribing circle d= 139 mm. Using pressboard wraps 1°5 mm thick as insulation between |v. winding and core, Inside diameter of L.v. winding=135+2 Xx 1'5= 138 min. Outside diam wer of | v. winding=138-4+2x9°l— HY, Winding ‘mm. Primary line voltage 11000 V. Connection =Delta Primary phase voltage Pp==1!000 V. ‘ <. Number of turns per phase T= 11000 x 111/250=- 486, As + 5% tapings are tobe provided, therefore, thy number of turns is increased to Mya 05% 4884—° 5128 The voltage per coil is about 1500 V..”. Using 8 coils. Voltage per coil=11000/8=1375 V. Turns per coilme5158/H-=641. Using 7 normal coils of 672 turns and one ceinforced coil of 424 turns Total h.v. turns provided Tp=7%672-+424=:5128. ‘Taking 24 layers per coil. Turns per layer Maximum voltage between layers=2 X28 x: limit. 2°25=126 V, which is below the allowable 25% 1000 Sx T1990 797 A. \ As the current is below 20 A, crossover coils are used for bv. winding Taking a current density of °4 A/mm? Area of hv conductor y=0 757/2°4==0,316 mm. Diameter of bare conductars=(4/m x 0°316)!*=0'635 mm. juctors. From Table 17°4 (IS : 3454— 1°66) the HV. winding phase current Ip=: Using paper covered cor standard conductor size has . bare «liameter: ‘63 mm, insulated diamerer=0 805 mm with fine covering. Modified area of conductor ap=/4 x (0°63)*=0 312 mm*. Actual value of current density used 8p=0°757/0 312—2'43 A/mmn?. Axial depth of one coil ==28 x 8°05=22°6 mm. The spacers used between adjacent coils are 5 mm in height. Axial length of h v. winding : Lep==number of coils x axial depth of each coil+depth uf spacers =8x226-+8X5=221 mm. . The height of wintow is 200 mm and therefore, the spice left hetween winding su window is 300—221=79 mm. This is occupied by insulation and axial bracing of the wa! ‘The clearance left on each side is 39°5 men, which is sufficient for 11 kV transformers, ‘The insulation used between layers is 0°3 mm thick paper. <. Radial depth of h.v. coil by=24 0°805 +23 X0°3= 26°22 mm. From Eqn. 7°22 the thickness of insulation between h.v. and L.v. winding is =540°9 kV=540°9x 112415 mm. This includes the width of oil duct also. ‘The insulation between h.v. and Iv. winding isa 5 mm thick bakelized paped cylinder. The h.v. winding is wound on a former 5 mm thick and the duct is 5 mm wide, making the toral insulation between h.v. and Lv. windings 15 mm. 436 ELECTRICAL MACHINE DESIGN Inside diameter of hv. winding re pee outside diameter Lv. winding-+2x thickness of insulation = 15624-2x 15= 1862 mm, - a Outside diameter of b.v. winding 26°2+2 x 26°92—=238'64 mm 239 mm. Clearance between windings of two adjacent limbs=255 -239=16 mm Repistance Mean diameter of primary winding= 19624299 012 mm. Length of mean turn of primary winding Lmte=" X212 x 10°3=0°666 m. Top Lmin @ Resistance of primary winding at 75°C rp= Ag x 0°21 2149 mm. 138+156°2 2 Mean diameter of secondary winding= Length of mean turn of secondary winding Lmu=m X 149 x 10-*=0°468 m. 111 x 0°21 x0 468 Resistance of secondary winding at 75' 149° =0°0732 0 4884 “Total resistance referred to primary side Rom2i9'2+(34.V" x0 0732-2364 . 0757x364_ PLU, resist 9751 %364_ 9095 + Leakage Reactance Mean diameter of windings=(138+239)/2= 188°5 mm. Length of mean turn Lge x 188°5 X 10° "592 m. Height of winding Lea Lull 2=(221+285)/2=263 mm, Leakage reactance of transformer refered to primary side Xpe2nf we THI ap Pet aant Te atts tes se2nx 50x40 10x canearx( = )x( 154 2622491 )x 10-2590 P.U. leakage reactance &4=0'757 x590/11000=0'0406. P.U. impedance €/=/(0°025)"+(0'0406}F=0 0477, Regulat P.U. regulation er cos $-+6s sin $. Per unit regulation at unity power factor €=€,==0°025, at zero p.f lagging = ts? 0°4°6, Ai 0°8 pF lagging €==0°025 x0 8+-0°0406 x 0'6=0:0444. Pe SaBINg Losses PR Loss BR loss at 75°C, = fs? Ry=3 X0°7578X 3645626 W. Taking stray load loss 15% of above. Total J*R loss including stray load loss Pe= 1°15 X626—720 W. Core Loss Taking the density of laminations as 7°6X 10" kg/m*, weight of 3 limbs=3 X 2°3 X0'010135 x 7°6 x 10°=69°3 kg ‘TRANSPORMERS be 431 ‘The flux density in the limbs is 1 Wb/m* and core lows is 1-2 W/kg esponding, to this density, specific oe Fig, 4°29 page 143 for 92 guns) “Core loss in limbs =369'3x 1'2=83'2 W Weight of two yokes=2 x0°624 x 012167 Corresponding to a flux denshy of 0 433 Whjin® ‘85 W. Core loss in yoke=115°3 X0'HS = W ‘Total core losses Pim=83'2+98'0 = 181 W. he 1 the yuke. the sp eitie core loss Efficiency, Total losses at full loal=181 +720-=901 W. ji . 251000 Efficiency at full load and unity pif =r mrag) , 114 0m ‘TRANSPORMERS 439 Windings HN. 1. Type of winding Gronover 2. Connections Delta 3. Conductor Dimention—bare 70x22 am? Diameter=0-63 mm insulated 15x27 mm* Diameter =0'816 au Area 149 mm 0312 mm ‘Number ia parallel None None 4. Curent density 223 Aja 2:43 A/mm* 5. Turns per phase in 4884 (5128 at—5% tap) 6 Coils total auraber : ax8 er core leg 1 8 7. Torme coil uw ur us et layer a ot era tras, of 4 ine 8, Number of layers 3 4 9. Height of winding 288 mm 221mm 10, Depth of winding 21mm 26mm 11. Tasulation Between layers 0-5 eum press board 03 mm paper Between coils 5-0 mm spacers 2 138 om 1862: mm Outside 1562 mm 239 mm 13. Length of mean turn 046m 0-666 m 14. Resistance at 75°C oon a 2192 @ 1, Between 1.v. winding and core ‘=prese board wraps 15 mm 2. Between Lv. winding and h.y. winding =bakelized paper 5-0 mm 3. Width of duct between Lv. and hy = =S am Tank 1. Dimensions ‘eight Hh 0:95im Jen th ly 035m width ™ oem 2 OillLevel onsm 3. Tubes Nil 4, ‘Temperature rise 319 Impedance 1, PU, Resistance ons 2. P.U, Resctance 040s 3. P.U, Impedance ous Losses 1, Total core lous 11 2, Total copper lom TOW 3. Total loses at full load so1W 4. Blliciency at fll load and vp. 965% 40 ELECTRICAL MACHIN! DESIGN Problem I. Design a 500 EVA, 50 Hz, 6600/400 V, single phase core type, oil immersed, naturel cooled power bansformer, The’ mean tempcrature rise of oll nt to eacerd C. Solution, Core De For this transformer we use 9+ Grade (G.K.W,) cold rolled oriented sie } lauunations. The following assumptions are inde : Ba=it5 Wh/m*; & Almm* j ge=8'9X 108 ky/m?; gi=7 6° X10® kg/m; ratio Lmi/le 0°5. ‘The values of constant K for various conditions have bewn workcd out in Example 7°10, page 404. The value of K is respectively 1°22, U' 745, 0375 and 0°344 for maximum efficiency 190% full load, minimum cost, minimui weight and minimum volume. [tis clear from above that if the transformer is designed for maximum. efficiency to occur at 90% full luad, the cost would be exhorbitant. ‘Ther fore, keeping the cost of the transformer the main consideration and keeping in view to some extent, the efficiency, volume and weight of transformer the value of K is assumed as 0 8, Voltage per turn Hal Q =0°84/50)=17-9 V. Flux y= 17°9/(4'44 x 50)=0'0806 Wb Net iron area As=0'0806/1'5=0'U537 m*=53'7x 10? mm, ‘A 3-stepped core is used. From ‘lable 7.3, for a 3-stepped core 1/0°6=300 mm. Apm=0'6 a? or diameter of circumscribing circle d= 537 I ‘The dimensions of the laminations are: a=0'9x300=270 mm, (Sce Fig. 7°53 page 382). ‘The standard sizes of laminations used are 270 mm, 215 mm and 135 mm. Window Dimensions. As the transformer is rated at 500 kVA, we take window space factor =0'7% 300=210 mm, ¢=0'42 x 300=126 mm. 10 io Kom sorEv + W466 From Eqn. 7°6, for a single transformer, Q=2'22 f Bu Kus 8 Au Aix 10~> 500—2'22 x50.x 1 5 X0'27X 2°75 x 10" Aw X 0°0537 x 107 Window area 4y=75'3X10¥ mm* ‘Taking the ratio height to width of window=2'5 . 25 Wat=75'3x 108. Width of window Ww=175 mm and height of window He=437°5 mm Distance between adjacent limbs D=d-+ Ww=300++175-=475 mm. Yoke Design ‘The area of yoke is taken tobe the same as that of core. Assuming rectangular cross-section for the yoke. Height of yoke Hy=a=270 mm and depth of yoke Dy=a==270 mm. Frame Height of frame=Hu-+2Hy=437'5-+2X270=977'5 mm. Width of framesD-+a=475+270=745 mm, Depth of frame 270 mm. Windings. LY. Winding LV. winding turns 7, 400/17-9=22'3 say 22. ‘Modified value of core flux density Ba=1'5X22°3/22=1'52 Wh/m?, 27. ‘TRANSFORMERS 441 LV. winding current (= 5201000 2 pon B current fo Gr Hs A Area of Lv. conductor a= —Sso~ 455 uit, Holic wis ding is used. ‘The L.v. condvctor consists of 20 strips connected in pa tach sip SEC ais Get wal ace Ta oe Rae Totel ara provided w= 20% 16H) mum, Actual current density used 8,=1250/468=::"67 .\/romé The arrangement of twenty conductors »s shown in Big 7°40 ‘The insulted conductors have dimensions 6°5X 4°5 mm. Axial depth of one turn=5X6°5-1-1 mm slack =35°5 mm. Raulial width of one turn=:5X4°5 +1 mea slack=19 min ‘The winding is applied on both the limbs with I turns per limb Therefore space has to be provided for V+] se, 12 turns. Axial height of Lv. comb Lea=12X33'5=402 ram. ‘The height of window is 437 5 mm and the balance height of 45°5 mm is occupied by end insulation and brs. cings etc. Radial depth or width of 1 v. coil ‘The insulation between core and Lv. winding is bakelite former 5 mm thick. Inside diameter of former d=300 mm. Outside diameter of former=3i0 mm. Inside diameter of L.v. winding=310 mm. Outside diameter of Lv. winding=310 +2 x 19 =348 mm Mean diameter of Lv. winding =(310+-348)/2=329 mm. Fig. 790. TV, conducts 03 m. Length of mean turn of lv winding Lmts=n x 0°329 Resistance of lv winding re 22£OTRI AIS Sooo a, FR losses in | v_ winding = (1250)* 0°00] =1562 W. HY. Winding LV. winding turns 7)=(6600/400) x 22=363. Using 182 turns on one limb and 181 on the other LV. winding current Jp= 509x000 =1575 4. Area of h.v. conductor ap=75'75/275=27'5 mm*, . ‘The available height of window is not sufficient for a single coil. ‘Therefore a double helical winding is used with, 91 turns in helix._ (On one limb there are 91 turns per helix but on the Other there are 91 in one and 90 in the other helix), «a ELECTRICAL MACHINE DESION The cross-section of bare conductor is 3°58 mm. It is wound on edj rith O° thick press board between turns. ‘s wound on edge with 0’ mm Area of hiv. conductor dp=27°4 mm. (see Table 17 1) Radial width of each coil= mm. Axial height of each turn=3'5+0°5=4 mm. | xia depth uf 91 turns 924 368 mm Insulation and bracing etc, mm Stuck 95mm Window Height (875mm Axial height of h.v. coil, Lrp=368-+9'5=377'5 mm, Insulation between h.v. and L.v. windings =5+0'9 kV=5-+40°9X 6611 mm. Using a duct 6 mm wide and a former 5 mm thick for first coil (helix) of hv. winding. Inside diameter of first hv. former=348+2 x 6=360 mm. Inside diameter of Ist h.v. coiloutside diameter of first h v. former =36042x5=370 mm. Outside diameter of first h.v. coil=370+2x8=386 mm. Allow a duct 8 mm wide including the former for second h.v. cuil. Inside diameter of second h.v. coil=386-+2x8=402 mm. Outside diameter of second h.v. coil=402+2X8=418 mm. Distance between h.v. windings on adjacent limbs=475—418=57 mm. Mean diameter of h.v. winding=(360+4418)/2=389 mm. Length of mean turn of h.v. winding Zmp=% x 389 mm=1'22 m. Resistance of .v. winding at 75°C; r= See oo 34.0 PR loss n h.v. winding ==(75'75)"x 0'34=1951 W.. Resistance Total IPR loss=1562+1951=3513 W. ‘This is is creased by about 10 percent to account for stray load loss. <1. Total JAR loss including additional loss Pe=1"1 x 35133865 W. :. Resistance of transf rmer referred to primary (h.v.) side 3865 se= GET HOE A. P.U. resistance of transformer t= Leakage Reactance Length of mean turn Lqumn SEM) 10-0114 m, ‘TRANSFORMERS: 4a Mean height of coil Le= “The windings are divuted equally over two limbs and thetefi re the leakage reactance of the transformer is equal to twice the leak ge reactance of windings irranged un one limb. Further the h.v, coil is divided into two parts and therefore Eqn. 7.71 is used. Leakage reactance of winding on each limb Kmtn foe TAP ( a Dmx 50x dex 10-7 x 1928x THE ( OHO OOU-o N+ 0 OLY Total leakage reactance of transformer referred tw primary Xp=2 x 0'982=1°964 Q. 75°75 x1°964 _ 9, P.U. leakage reactance es 0025, P.U. Impedance e= 0 0082*4-0 02 Lor 0°0236. PR loss, Total copper loss Py=3865 W. Core toss, As inentioned earlier 0 33 mm thick 56 ;,ade (G K.W.) laminations are used Length of mean flux path {s=2(487°5+475 +270) x 10-—2'37 10 Weight of irona2'37 x 0°0537 x7 65 X 108=974 kg. The specific loss tor Ba==1'32 Wh/mt, from Fig, 4°26, is 1-25 W/ks, Core loss=1°25 x 974=1217°4 W. Making an allowance of 20% for joints, total core loss Pos] 2x 121751460 W. Total loss at full load=3865+ 1460=5325 W. Efficiency Efficiency at full load and unity p.f. 500x108 geo) BOOK 10 FSET * 10985 % Regulation Regulation at full load and 08 p.f. lagging e=er cos $+ es sin ¢, =0:0077 X0'8+.0'0225 +0°6=0°0197 p.u. or 19°7%, Tank Height of frame H=977'5 mm. Allowing 50 mm for the base and 250 mm for oi! ‘Oil level=977°5+50+250a11300 mm. Allowing 300 mm further for leads ete. 44g BLECTRICAL MACHINE DESIGN pa 0 00000000000 O00000000000 Height of tank f = 1600 mu Allowing a Qearance of 80 mm at each along the length, 1 of tank Lae 475-4 418 $200 mma 105 m Acclearance of about 200 mm is allowed ulling the width ty accomm- «0 Width of tank We==418420) mmeeh'62 m Loss dissipating surface of tank Be=QM1-U5 | 0°62) x16=5 44 nt "5 The mean temperature rise of | OOOOOQOQOOOOO0O | | oil is to be limited to 35°C. OOOOOO000000 Let the acea of tubes be 25. = 050 2. Specific loss dissipation 5325, 285 “53a tangs “ise Wi WS _ 1254882 x 91. Core and winding details of $00 kVA, jingle Phase 6600/400 volt, 50 Hz core type ower transformer (dimemsions in mm) or (see Eqn. 7.8) 2, r= 182 72 m* fe Atea of tubes needes'=> 182% 5°34 Using 50 mm diameter tubes spaced 75mm apart, ‘The average length of tubes i= assumed ais 1°35 m, Dissipating ates of each tubes 005 x 135=0-212 mt =46, Along length—: ws 12 and 1 tubes ils of core warstingy and tank are given in Fig. 791. Probleme IIT. Ji sign 150 kl, 6600/4001", 50 Hz, single phase ahell type, vil smmersed, elf cooled purr transformer. | Average teriperature rise of oil not to exceed 35 Solution, Core Desiga, roi taken as |. vo Veltags per turn Be Ev 19-9 44d f~ 44x50 Taking the valuc of maximum flux density Ba=1'25 Wb/m‘. Net iron area 4¢=0°0552/1'25=0°0442 m*==44'2 x 108 mm? ‘Taking the iron stacking factur=0'9, Gross iron area Agi=44'2/0'9e=49 x 10% mm!, The ratio of 8/24 is taken as 25 (See Fig. 771). s+ 25(2a)%=49x 108 or width of central limb 2a=140 mm, Depth of frame 6=2°5x140=950 mia Table 7"1, the value of output coefficient K is 1x 150= 12°25 V. Flux n=: =0'0552 Wb. ‘TRANSFORMERS 445, The side limbs carry half of the flux and Coen fove uhein wid) eqn. to half of th width of c ntral limb or width of side limby a==70 mm Window Dimensions. lo lo PRY GE Window space factor Ke > For the sake of calculation of window area, a current d assumed. Now, QH222 f Be Bu B As Aw 10° or 350m 2°22 x 50x 1°25 0°27 X2°3 % 108 A044 A bee 1, 2 Window area Ay=0'0394 m?m=39°4 x 10° aim? The height of the window is taken as twice th: width of window, “or 2We?: 2. Width of window=140 mm, and height of window M280 mm. y of 25 Afmm? is Yoke Design, The flux in the voke is half of the flue in the central limb. The flux density in the yoke is taken same as that for limbs. Height of yoke Hy=a=70 mm, Frame. Reference Fig. 7°92. Height of frame H=He-+2Hy=280-+2 x 70=420 mm. Po 420 Fig. 792. Prams of shell type (ransformer Fig. 793, Arran ‘ ‘ (All dim asions ia'mm) (Ati dimeasions nim) 446 ELECTRICAL MACHINE DESIGN Width of frame 1 ~4a-+2 Wed x 20-42 x :40= 560 mm, Depth of frame Dy=b= 50 mm Windings. Number of turns in Ly winding 7.=440/12 950 36, This number is divisible by four to allow the subdivision of Lv. winding into 4 sections 01 4 turns each. Number of turns in the h.v_ winding 77p=36 x 6600/440=540, This ix divic io two coils of 150 turns each and two of 129 turns cach with reine forced end turns. The arrangement of the coils is sh wn in Fig. 793. The main insulation, height wise in the window is givin below : Tnwulation between cnd Lv. coils and cure=2 x 1U= 20 mm, Spacers between hw, and Lv. coils=4x20=80 mm. Spncers between Ly coils=1x8=8 mm. Spacers between h.v. coils=2x10=20 mm, ‘Total height of insulation=20-+ 8048 +20=128 mm. *. Height available for conductors=280—128= 152 mm. Total number of h v. and Lv. cois=8. ., Height of each coil=152/8=1) mm, HV, Winding, HV. winding current Ip= 450% 1000 S227 A, Area of h.v. winding conductor 6=22'7/2'3=9'9 mm, The h.v. conductor comprises of two paper covered strips 8X 0°6 mm bare in parallel Arca of hv. conductor used ay=2*8X0'6=9'6 mm?. Actual value of current density used in h.v. winding=22°7/9'6=2°37 A/mmt, Space taken by h.v. conductor in a coil height wise=2X8=16 mm. The balance height (1°—16)=3 mm is taken up by paper. Space taken by h v. bare conductors width wise=150x0'6=90 mm, The space taken up by insulation between h.v. and core width wis Space for winding along width of window=140—40=100 inm. or width of b.v. coil=100 mm. The balance spacess100—90—=10 mm is taken by insulation between h.v. conductors, L. V. Winding x20=40 mm. L. V. widing current Z='0% 1000341 A, Area of Ly. winding conductor aye=-Stl = 148 mm*. Using 4 strips of 7°55 mm each in parallel. Area of I.v. winding conductor used a.m=4X7'5.5—150 mm?, Actual value of current density used in Lv. winding &=341/150=2'28 A/mm?, Space occupied by bare conductors in a coil height wise 2X7 5=15 mm, The balance space (19—15)=4 mm is taken by insulation ete. Space occupied by bare conductor, in a section, width wise =29X2X5=90 mm. ‘TRANSPORMERS: 447 Using 1 mm thick press board width wise between each conductor. + Width of Lv. coil=94-9 x 1=¢ 100 mm. ‘The balance space 140—100=40 mn» along the width is taken up by insulation bet. ween core and I.v. winding with 20 mm on each side. Resistance. The approximate length of mean turn=Lmip= Lm —VE8 m. 0-021 540 1°58 Resistance of h.v. winding r= = 1870. Resinance of Lv. winding r= °M21 53671 9.o. yim a. Resistance of Lv. winding referred to h.v. side ry’ =°0038 (54, 36)*me1 8 42. Total resistance of transformer refvrrcd to primaty side Rp==l 87+ 1U=3'67 4 =3°85 © (taking loss in connections ete.) PLU, resistance e¢= 227X385 .-9133, Leakage Reactnnce. For sandwich coils, lezhage reactance of transformer refetred to primary “Te be Xp=nf oy a nel apeth ) (See Eqn. 7°80) 58 540% 2 mm 50 de 107 LE 5 ( oo2ss4 Leeerooe )=1730 PLU. reactance 64= 727217320 9595. PU. impedance VOTRE 0595%0 O61 Regalation, The regulation of transformer at full loa! and 0°8 pf a ssing. =e cos $-es sin § =0'0125 X 0'8-+0°0595 X06 -=0 0457=4 57", Losses TPR Loss TAR loss in h.v. winding =22' 7" x 1°87=964 W. PR loss in Iv. winding=341? x 0'008=228 W. Total FPR lossm964+928= 1892 W, ‘The total I®R loss including loss in connections etc. may be taken as Pe 2000 W. Core Loss. Referring to Fig. 7°98. ‘Volume of iron=14X 35 X28-+2 x 7X 35x 56-+2 X7 x 35x 28= 0'05488 m* ‘Weight of iron=0°05488 x 7°6 x 109=417 kg. ‘The specific iron loss for a flux density of 1°25 Wb/m* is 1°8 W. ‘Total core loss=417 x 1'8=750 W. The total core loss including additional losses may be take UXiciency Total 10. at full load=2000-+840=2840 W 150 x 10! se , 15010" 99 98997 Ficiency at full ivad unity power factors 55" ioyaqy * 100—98'3% P.U. load for maximum efficiency==¥ 840/2000—0°65 Toad for maximum efficiency=0°65 x 150=97°5 kVA. 840 W. 408 ELECTRICAL MACHINE DESIGN DESIGN OF SMALL SINGLE PHASE TRANSFORMERS 7-61 Introduction, he design of small low voltage transformery of ratings from 10 to 1600 VA is bring given here in order to help the students in tiveir prj ct work. 762, Core De ‘The starting point in the design of small transformers is the choice of turns per volt, values of turns per volt are given in Table 7-4, Table 7-7, Turns per volt T,, vA Tums per volt vA Turns per volt 0 23 200 3s 1s irs 250 28 2» wo 300 28 2s 7 400 2 0 10 500 20 s 56 750 17 100 46 1000 16 130 40 | Now K=4°44{% 7 2°. Turns per volt T4=7/E=1/4'44 f On. Flux in the core m=1/5 44 f Ts. The frequency of the transformer is specified and the value of turns per volt 7's is taken from Table 77. Therefore flux ®m in the core is known, Net area of core A= @q/Bm The value of maximum flux density Bu is assumed to be 1 Wb/m?, Gross area of core Api=Ay0, (Assuming a stacking factor of 0°9) A shell type of construction is normally used for small transformers. ‘The core is made up of any of the following combination of stampingy. (i) Band I (Fig. 794(0)), (6) Tand U (Fig. 7-946) (iii) used in pairs (Fig. 7-94(c)). L Oh RAS OR SSS RTT La LU C b-—» —4 f (a) El type. (6) T-U type. (©) Btype Fig. 794, Laminations for small transformers, . Tables 7.0, 7°9, and 7°10 give information about standard stampings manufactured by Pre... ion Pressing Division of M/s Guest Keen Williams for small transformers and chokes. ‘TRANSPORMERS, “ Table 7:8, E—I Stampings [(Fig. 794(a)) nn ee Dimensions | No. 4 B c Remarhs my | oe yr | yar | ae a 12a | sie rer | rete | sper | site a | se r aye | spor 38 10 | 58° aye | ze | per ae wa | se 238° a8" oe 1 ayszr | ayer sie Si m™ June | rane ase | saree B 24M" 38° 318" u ¥ 318 38" nA x 38 38° 2 x 38° sre 30 6mm | somm | 10mm | 10mm 4 hotes $/32° dia. n zs | ayer | 76 m6 4s 2ser | aang | 76" m8 4 holes 5/32" dia. 1” Yr aae | ane ue 4 holes 7/32" dia. “opr x aap | aa a ‘A boles 7/32" dia, wie sso | 2s | ame | ape ‘A holes 7/32" dia, 4 v suse | signet | ame) ae 4 holes 7/32" dia. 3 | 28mm | som | mm | om | 4mm A oles 11/64" dia, 3 riya x3" se" se se 4 holes 7/32° dia. 3} rayar | ae sant | ine ape A botes 7/32" dia. 4a | vse | ayer | sane | ays | aye A holes 3/16" dla, 16 vue | aa | same | 3 “ 4 holes 732" dia. 5 rye | ase | amr | ee 0° A holes 17164" dia, 6 Jue |e aap | ee “" 4 holes 9792" dia. 7 |e © aisne | r 4 hole 17164" dia, a |e ° s v ¥ A holes 17164" dla, s fa mye | ese fe v A doles 1/8" dia. SS SLECTRICAL MACHINE DESIGN Table 79, T—U Stampings [(Fig. 7:94(0)]. Irmensions No. | E Remarks =; ———-—- —— “ bape og 4 holes 5/92" 9 yore | ane 4 holes 7732 dia 9A [ane | ane 4 holes 62° di, ax nr | m6 A hake 532° dia, is Lae i 4 Uholes 1/4" dia. 3a 3a" 4 holes 1/4" dia, Table 7 10, E ~E Stampin, i(Pig. 7 940), Dimensions a] 07 ons | oars: 32° dia hoe 079" 08 | 585" 0-485" oars 4-332" dia holes | : i \ 1--3/16" dia hole Asquate section is normally used for the central limb, i.e. the depth of the core is marie equal ws the width of the ceutzal timb or width of cratral ‘A standard stamping giving a width A nearly equal fo the value calculated above may be used 763, Winding Desiga, Current in the primary winding Jp=VA/n. “The «fficiency of small transformers varies from 80 to 96 per cent. ‘Arca of primary winding conductors ap='Iy/% mm, where is the current density in the primary winding conductors in A/mm. A value of 2°3 A/mm? may be used. Enamelled round conductors are used for the windings of small transformers. ‘TRANSFORMERS asi chapted’ 2" 24 ze of conductor is selected by referring ‘© erlevant. tables given in apter ‘Turns ia primary winding 77=Fp7s, Current in secondary winding L=VA/Fs Area of seconclary winding conductor 0,=,/8. mnt, When calculating the number of secondary winding surns, an allowance of 5 percent exrta turns is made to compensate for the voltage drop it. the windings Secondary winding turns =1°05 VT 7 4, Window Ar (i) primary winding (ii) secondary winding (ii) insulation and the former (bobbin) on which the windings are supported. Space is required in the window for : Space required for primary windings Tate Space required for swcondary_ windings = oF Space factor 3=0 8 (d/dy)* where d=diameter of bare conductor and d,=diameter of insulated conductor, ‘The space required for insulation and former is estimated as 20% of that required for the windings. Window area requires 12 (window area required for primary and secondary windings). Tt should be checked that the stamping used, gives a higher value of window area than the value calculated above, Design Problem IV. Design a single phase tranaformer to be connected to a 280 V, 50 Hz supply. The transformer ia to deliver 3A at 50 V. Solution, Gore. Volt ampere rating of transformer=50 x3=150 VA. From Table 7-7, turns per volt Ta==4'0. ! 1 21125 m Wi Om FGA ITs TUK GOKE Wbe1"125 m Wb. Taking a flux density of 1°0 Wb/m*, . 1. Netiron area of core, Avr 2 = 12BXIC™ ytont-125 x10* mm Gross core area Api==1"125X 10°%/0°9=1°255 108 mm? ng a square section for the central limb. Width of central limb A=n12°5x 10*=35°4 mm, Primary winding, The eficency of this transformer is assumed ax 92%. mary wi YA, 10 m0 A Primary winding current Ip= = Gap camp ‘Taking a current density of 2°3 A/mm*. on Area of primary winding conductor opm jrg-=0'809 mm*. 452 BLUCTRICAL MACHINE DFSIGN Diamen of bare conductor=0'626 mm. Using enameled conductors. From Table 17°7, the nearest _standurd conductor has baccdiawter=0.6. mm ‘The diam«ter of insulated cbnductor is 0°707 mm. Sp ice factor for primary winding =0'8(0'63/0"707)=0°635, ‘Area of primary conductor used =F 63)'=0'312 mm. Number of primary winding tnrns T=VpT'e=230% 4=920, ace reauite imary winding 22079200312 4a Window space required by primary winding==5? =A 57 meas mni*, Secondary winding. Sccondary winding current I,=3 A. Aiea of secondary winding conductor a=35,s=1°3 mm*, ameter of bare conductor= 1285 mm. enamelled conductor for secondary winding. arest standard conductor has bare diamcter=1'32 mm. diameter of insulated conductor=1'42 mm. Space fuctor for secondary winding =0'8 (1°32/1°42)'=0'69, Area of secondary winding conductor any (1'32)'=1°37 mm, Number of secondary winding turns 7'4=1°05 VsTe=1°05x 50X3=210. Window space required by secondary winding Bete OATS 17 mm. Stamping Size. ‘Total window space required Aw=1'2 (space required for primary and secondary winding) 2(452-4+-417=1045 mm* ‘We have now to select a stamping which gives : width of central limb 4=35'4 mm=1"39" and area of window 41045 mam*= 1°62 sq. inch. Using a combination of E and I stampings, From Table 7°8, seclecting No. 16 Azali", B=4h’, C=3'3/4", D=m3/4", Bom3/4". B~A-2D z From Fig. 7°94(a), width of window We=- =3/4", and height of windew Hy=O—2=23" Area of window provided Aw= WwX Hum3/4" X2$=1°69 sq. inch. Ts1sis more than the réquired window space. Width of central limb 4=1}"=38'1 mm, UNSOLVED PROBLEMS 1. Cateuate the core and window ares required, for 1000 kVA, 6600/400', 80 Is single phase core tyre trinaformer. Assume a maximam flux density of 125 Wb/m* and a current density of 25. A/mm. Voltage per turnsn30 V, and window space factor=0'32. Ame, 108510" mm®; 83-4 10" ma] 2. A3 phase, 50 Hz, oil cooled core type transformer ‘has the followin: dimensions : distance between gore centres~-O-2m ; height of windows0-2 m; diameter of circumscribing circle=0'l4 m. The fux ‘density in the core is'1-25 Wb/m* and the current density in the conductors is 2°5 Aymm*. Estimate the kVA Feting- “Autumn a window specs fctor of 2 anda cor re faetor—=0'6."The core is 2 opped. ‘TRANSFORMERS. 45s 3, Determine the dimensions for core aad yoke for aS kVA. SOHz, single phase core type transhae. mer. “A rectangular core is used with lovg sidp twice as long us short side. ‘The. winstow height is 3 ated the width. Volta: e per turn is 18 02: current density 18 A/mm? : fine decoriy 1 Wb Tans. 4¢=8100 mm rim? ; Ags 15 $x 10 mon! window 72 5.2073 maah 4. Estimate the main dimensions ncludine winding conductor areas of a3 phi: delta/star core type transformer rated at 300 AVA. 6600/40 V. $0. HA. suabe cre. with three steps Hving a itcummeibing Gircle of 0:25 m diameter and'a leg spacing of O-4 mis available The emf ner turns S*V. Assume a currea density of 2:5 Almm?. a window space factor of 0-28, and a stacking factor uf 09 [Ans. 7,=716, ayr=6 min? ; Ty230, aym157-5 mm*; window 0 15 «0 45 m? scores 47'S x 10* mma!) 5. Determine the dimensions of the core, the umber of turns and the cross-sectional area of conduc tora in the primary and secondary windings of a QD KVA, 22001480 V sin fe phase core type ranafor mer to operate a Hz, assuming the followin: data: approximate voltage per turn, 7:5 Vi maximum flux io, of etective erosnectional arca of core to square of diameter i ight 10 width of window, 2 ‘eorrent deni [Ans 2 amd 6. Calculate the dimengiogs of the core the number of turns and the crow sections of the conductors for» 100 KVA, 2300 400 V, 30 Fin ‘phase shell type transformer, assuming : atic af rmgnetic and electric, Tull Towa), 480.°10°*; maximum flux density I-] Wim; current 22x M space factor, 0'3: ratio of depih of stacked core to width of central limb, 2°6 : ratio of height to width of window, 2°5 :stacking factor 0-9. [Ams. 135 340 mm? ; 230, 40; 19°8 mm, 114 mm#] 7. Two single phase transforiners with AN cooling having linear dimensions in the ratio x: 1, are designed to work at the same current density, fux density and frequency. Calculate relative ratings, lossee and total weights per kilovolt arapere of the two tranformers Comment briefly on the temperature rise of the frst transformer for increasing values of, sasuming that the second transformer operates with permissible temperature rite on full load. (Ans a8: iat; tarts A) 8, A750 KVA, 6600 V, 50 Hz, three phase, delta/star core tyne transformer has the followine dada winding, 30 idth of h.v. winding, 25 mmm, width of dust Between h.v. and Lv. windia height of windings, 0-4 m: length of m'an turn 15m, h.v. ‘vindiog turns, 217. Estimate ths reactance af the trancformer refered (o/h v- ade. Eatimate the per unit regulation of the transformer aifulfiond and 08 power fustor laine ifthe reistance per phase reterid to te hv. side B08 a Tans. 232 8, 00347) 9. Derive an expression for the leakare inductance per limb of the windings of a single phase trans former referred. to the primary side in terms of the relevant quantities, and calcutate this inductance ia given case where the windings have a mean length of turn I'S'm and cach windine has a radial depth So'mm. ‘The radial clearance between windings on the same limb is 35mm and. the axial height of each, winding is 0% m, The primary winding consists of 240 turns per limb. (Ans. "8 me Ina transformer having a 10 : 1 turns ratio, the copper loss in the primary winding is 15 per cent than that in the secondary winding. The resistance per turn of the primary is 0°00394 &. and there arc I more turns in the primary than in the secondary. Calculate the resistance of szcondary winding. Ans. 0-0675 a] 11. A 100 kVA, 2000/400 V_$0 Hz, single phase shel! type transformer has two full b.v. coils, one full, coil and two half Lv. coils. Calculate the value of average axial force acting on the end coils under short cireutt IC full voltage if its leakage reactance is 0-035 pu. Length of mcan turn 1'5m, width of coils, O12 winding turns, 200 ; p.u. resistance of transformer, 0-01. Ant. 35°23 10 Ni 12._A 20/110 V, 1kVA, $0 Hz single phase transformer has a core witha uniform crom sectional - arca of 2500 mm*, an effective magnetic core length of 4 mand acore weight of 8 kg. If the core is worked: of 12 Woim', the correspondine ma’netizing force, H, is 700 Afm and. the pecie core, ow determine (@) the transformer no load current when tien v. ied st Ov, the corresponding magnetizing reactance and the equivalent shunt resistance to represent orl ™ "* (Aas. O17 A: 1287 8 £6600 a} 13.__A 300 kVA, 6600/440 V, three phase delta/star core tyne transfirmer hasa maxitnum flux density of 1-35 Whim and the total weight of core in 650 kx. The magnetizing VA/kg and the iron losvkg. corres: ponding to 1:35 Whim? are 30 and 2°5W sespectively. Calculate the no load current if the mmf required fog joints is 2°S percent of that for iron. (Aus. 0 695 A pec phase] 14. The tank of w 1250 KVA natural oil cooled transformer has the dimensions length, width and deight as 1°55 mx 0°65 mx 1-85 m ively. ‘Th fall fond lomris 137, KW. ‘Pind the number of tubee for ‘his transformer astuming : Wim'_"C due to radiation=6 : Wim'—*C due to convection =6:5 : improvement convection dus to provision of tubes=40 per cent ; temperature ritem40°C ; leagth of each tube I a5 fiameter of tuber 50 mm. Neglect the top and bottom surfaces of the tank as revards cooling. saa 190 tben] tt 15, With 100 A taken from the terminals ofa certain transform:r connected to a os poe xppy cheat the realting otal reactive drop i$ psx cont. Calclat the poreentage reactive Wrrnand wib tS sr cont ose ras in on windinas ani. Conected to'a 60 He Grout wit dad drawing 110 A from sscondiry winding. Asturas the terminal voltag: a1 th: dimon sions of all aaths to remal unalte; (Ans. 17'S per cent]

You might also like